Ev.- Maternity and Women's Health Nursing

Réussis tes devoirs et examens dès maintenant avec Quizwiz!

The school nurse is teaching a group of 16-year-old girls about the female reproductive system. One student asks how long after ovulation it is possible for conception to occur. The most accurate response by the nurse is based on the knowledge that an ovum is no longer viable after:

24 hours The ovum is viable for about 24 hours after ovulation; if not fertilized before this time, it degenerates.

A primigravida with type 1 diabetes is having her first prenatal visit. While discussing changes in insulin needs during pregnancy and after birth, the nurse explains that in light of the client's blood glucose readings she should expect to increase her insulin dosage. Between which weeks of gestation is this expected to occur?

24th and 28th weeks of gestation

A newborn of 30 weeks' gestation has a heart rate of 86 beats/min and slow, irregular respirations. The infant grimaces in response to suctioning, is cyanotic, and has flaccid muscle tone. What Apgar score should the nurse assign to this neonate?

3

A nurse in the clinic determines that a 4-day-old neonate who was born at home has a purulent discharge from the eyes. What condition does the nurse suspect?

Chlamydia trachomatis infection Chlamydia trachomatis infection occurs 3 to 4 days after birth; if it is not treated prophylactically with an antibiotic at birth or within 3 days, chronic follicular conjunctivitis with conjunctival scarring will occur.

A practitioner orders doxycycline (Vibramycin) for a sexually active woman with a history of a mucopurulent discharge and bleeding associated with cervical dysplasia, dysuria, and dyspareunia. With which sexually transmitted infection are these clinical findings and medication therapy commonly associated?

Chlamydial infection The signs and symptoms listed and the treatment ordered (doxycycline or azithromycin) indicate that the client has a chlamydial infection.

A nurse is teaching a class of expectant parents about changes that are to be expected during pregnancy. What changes does the nurse explain result from the melanocyte-stimulating hormone? (Select all that apply.)

Chloasma Linea nigra

A client who is in labor is admitted 30 hours after her membranes ruptured. For what condition does the nurse anticipate that the client is most at risk?

Chorioamnionitis

A client who is in labor is admitted 30 hours after her membranes ruptured. For what condition does the nurse anticipate that the client is most at risk?

Chorioamnionitis The risk of developing chorioamnionitis (intra-amniotic infection) is increased with prolonged rupture of the membranes; foul-smelling fluid is a sign of infection.

During labor a client begins to experience dizziness and tingling of her hands. What should the nurse instruct the client to do?

Breathe into her cupped hands Dizziness and tingling of the hands are signs of respiratory alkalosis , probably the result of hyperventilating; breathing into cupped hands promotes the rebreathing of carbon dioxide.

A nurse is assessing a newborn. Which sign should the nurse report?

Breathing pattern with recurrent sternal retractions

A nurse is assessing a newborn. Which sign should the nurse report?

Breathing pattern with recurrent sternal retractions This infant's breathing pattern is indicative of respiratory distress ; the expected pattern is abdominal with synchronous chest movement.

The parents of a newborn are told that their neonate may have Down syndrome and that additional diagnostic studies will be done to confirm this diagnosis. What procedure does the nurse expect to be performed?

Buccal smear The cells in the buccal smear provide a pictorial analysis of chromosomes and show chromosomal abnormalities such as the trisomy found in Down syndrome.

A nurse who is monitoring the blood glucose level of the term infant of a diabetic mother (IDM) identifies a blood glucose level of 48 mg/dL. What should the nurse do?

Continue to monitor the blood glucose level per policy. A reading of 48 mg/dL is within the expected blood glucose range for a neonate (40-60 mg/dL) and requires no measures other than continued monitoring for the next 24 hours.

A client has a child with Tay-Sachs disease and wants to become pregnant again. She tells the nurse, "I'm worried it will happen again." How should the nurse respond?

"Have you considered the option of genetic counseling?" A rare, inherited disorder that destroys nerve cells in the brain and spinal cord. Although the disease is rare in the general population, it is an inherited autosomal recessive disorder and there is a 25% probability that it will occur again in the same family.

On her first prenatal visit a client says to the nurse, "I guess I'll be having an internal examination today." What is the nurse's best response?

"Have you ever had an internal examination done before?"

A nurse is giving discharge instructions to a client who has undergone anterior colporrhaphy. The nurse knows the teaching has been understood if the client says:

" For discomfort I can try using warm compresses on my abdomen."

A client at 31 weeks' gestation is admitted in preterm labor. She asks the nurse whether there is any medication that can stop the contractions. What is the nurse's response?

"A beta-adrenergic." Beta-adrenergic medications are tocolytic agents that may halt labor, although only temporarily.

A 35-year-old client is scheduled for a vaginal hysterectomy. She asks the nurse about the changes she should expect after surgery. How should the nurse respond?

"A hysterectomy doesn't affect the chronological age when menopause usually occurs."

A nurse on the postpartum unit discusses breast care with a client who is formula feeding her newborn. Which statement indicates to the nurse that more teaching is needed?

"Applying heat to my breasts will help ease the discomfort."

A mother asks the neonatal nurse why her infant must be monitored for hypoglycemia when her type 1 diabetes was in excellent control during her pregnancy. How should the nurse respond?

"Babies of mothers with diabetes have a higher-than-average insulin level because of the excess glucose received from the mothers during pregnancy, so the glucose level may drop."

While teaching a prenatal class about infant feeding, the nurse is asked about the relationship between breast size and ease of breastfeeding. How should the nurse respond?

"Breast size is not related to milk production."

A nurse is teaching a childbirth class to a group of pregnant women. One of the women asks the nurse at what point during the pregnancy the embryo becomes a fetus. How should the nurse respond?

"During the eighth week of the pregnancy."

A nurse teaches a client with asthma about her illness during pregnancy. Which statement by the client indicates that the nurse's teaching has been effective?

"I can use my albuterol inhaler if it's absolutely necessary."

After an uneventful 8-hour labor a client gives birth. Once the airway has been ensured and the neonate has been dried and wrapped in a blanket, the nurse places the newborn in the mother's arms. The mother asks, "Is my baby normal?" What is the best response by the nurse?

"Let's unwrap your baby so you can see for yourself."

A client in active labor is rushed from the emergency department to the labor and birth suite screaming, "Knock me out!" Examination reveals that her cervix is dilated 9 cm. What should the nurse say while trying to calm the client?

"Medication may interfere with the baby's first breaths; keep breathing."

A nurse is teaching a woman how to perform breast self-examination. Which statement indicates that the client needs further education?

"My breasts are so tender right before my period that I hate doing it."

A client receives spinal anesthesia during labor and birth. Twenty-four hours later, she tells a nurse that she has a headache. Which statements indicate to the nurse that the headache is a reaction to the anesthesia? (Select all that apply.)

"My ears are ringing." "It gets better when I lie down." "Bright lights really bother my eyes." "My head hurts more when I'm sitting watching TV."

A client at 16 weeks' gestation is being treated for Trichomonas vaginalis. Which statement best indicates to the nurse that the client has learned measures to prevent a recurrence?

"My partner has to get treated before we have sex again."

A client who is breastfeeding tells a nurse that her breasts are swollen and painful. What can the nurse teach her to do to limit engorgement?

"Nurse at least every 3 hours for at least 10 minutes on each breast."

A couple arrives at the newborn nursery asking to take their newborn grandson to his mother's room. What is the best response by the nurse?

"Please go on to see your daughter. I'll bring the baby to her room."

The nurse concludes that a couple with a newborn with Erb's palsy has an accurate understanding of the infant's prognosis. Which statement confirms this conclusion?

"Recovery usually occurs in about 3 months."

What instruction should a nurse include when teaching about the correct use of a female condom?

"Remove the condom before standing up."

A client has just given birth to an infant with Down syndrome. The mother is crying and asks the nurse what she is supposed to do now. What is the nurse's best response?

"Tell me what you know about Down syndrome."

A pregnant client tells the nurse that she thinks she has developed an allergy because her nose is often very congested and she has difficulty breathing. How should the nurse reply?

"That is an expected occurrence; the increased hormones are responsible for the congestion."

A pregnant client in the third trimester tells the nurse in the prenatal clinic that she has heartburn after every meal. What explanation should the nurse give about the cause of the heartburn?

"The cardiac sphincter relaxes and allows acid to be regurgitated."

The mother of a newborn son tells the nurse that she is concerned about circumcision because of the pain involved. What is the nurse's best response?

"The health care provider will tell you how your baby's pain will be controlled."

The family of a pregnant client with myasthenia gravis asks the nurse whether the client will be an invalid. What is the best response by the nurse?

"The progression is slow, so people with myasthenia will spend their younger life with few problems."

A mother is inspecting her newborn girl for the first time. The infant's breasts are edematous, and she has a pink vaginal discharge. How should the nurse respond when the mother asks what is wrong?

"The swelling and discharge are expected. They're a response to your hormones."

During a childbirth class, several participants have questions about the elective induction of labor. One participant states that it is more convenient for a woman with a busy schedule. What evidenced-based information should the nurse provide to the participant?

"The widespread use of elective induction increases the risk of unfavorable outcomes."

On the first day after a mastectomy, a nurse encourages the client to perform exercises such as flexion and extension of the fingers and pronation and supination of the hand. The client asks why she has to do these exercises. The best response by the nurse is:

"They will help stimulate peripheral circulation."

The nurse is caring for a couple after the birth of their first child. What should the nurse tell the family to do when their infant is exhibiting the behavior demonstrated in the picture?

"This is the time when the baby is likely to be most responsive to you."

A client at 37 weeks' gestation gives birth to a healthy boy. While inspecting her newborn in the birthing room, the client becomes concerned and asks, "What's this sticky white stuff all over him?" How should the nurse respond?

"This is vernix. It helps protect the baby while he's in the uterus."

During a follow-up appointment, a client at 21 weeks' gestation is found to have hyperemesis gravidarum. The client says, "Why is this happening to me? I don't know whether I can continue like this." What is the best response by the nurse?

"This must be physically and emotionally challenging for you."

A client arrives in the birthing room with the fetal caput emerging. What should the nurse say to the client during a contraction?

"Use the panting-breathing pattern."

During a prenatal visit a client who is at 36 weeks' gestation states that she is having uncomfortable irregular contractions. How should the nurse respond?

"Walk around until they subside."

A client who is pregnant for the first time and is carrying twins is scheduled for a cesarean birth. What should the nurse tell the client to expect?

"We'll be encouraging you to walk early after surgery."

A nurse reviews the prescribed treatment with the parents of an infant born with bilateral clubfeet. Which parental statement indicates to the nurse that further education is required?

"We'll have to have the baby fitted with prosthetic devices before he'll be able to walk."

When encouraging a client to cough and deep breathe after a bilateral mastectomy, the client says, "Leave me alone! Don't you know I'm in pain?" What is the nurse's most therapeutic response?

"What's your pain level right now?"

An older female client tells the nurse in the clinic that she has a cystocele that was diagnosed a year ago. She has urinary frequency and burning on urination. The client asks, "The doctor wanted me to have surgery for the cystocele last year, but I can manage with peripads. It won't hurt not to have surgery, will it?" How should the nurse respond?

"Yes, you're risking kidney damage."

A multipara who is admitted to the hospital for repair of a rectocele and cystocele asks a nurse why these problems happened to her. How should the nurse respond?

"You have relaxation of the muscles in your lower pelvis." Relaxation of the pelvic musculature causes the uterus to drop, with subsequent relaxation of the vaginal walls, most often as a result of childbirth. A rectocele is a protrusion of the rectal wall into the vagina, whereas a cystocele is a protrusion of the bladder into the vaginal wall.

A woman visits the prenatal clinic because an over-the-counter pregnancy test has rendered a positive result. After the initial examination verifies the pregnancy, the nurse explains some of the metabolic changes that occur during the first trimester of pregnancy. (Select all that apply.)

Sleep needs increase. Fluid retention increases. Calcium requirements increase.

A nurse concludes that a positive contraction stress test (CST) result may be indicative of potential fetal compromise. A CST result is considered positive when during contractions the fetal heart rate shows:

Late decelerations The fetus with a borderline cardiac reserve will demonstrate hypoxia by a decreased heart rate when there is minimal stress, making the CST result positive.

A client gives birth to a baby weighing 7 lb 2 oz and decides to breastfeed. The nurse is instructing the client regarding breastfeeding. What should the nurse tell the client to expect?

Lochial flow will be increased.

A client gives birth to a baby weighing 7 lb 2 oz and decides to breastfeed. The nurse is instructing the client regarding breastfeeding. What should the nurse tell the client to expect?

Lochial flow will be increased. Breastfeeding stimulates oxytocin release and uterine contractions, resulting in increased lochial flow.

During the first hour after a cesarean birth, a nurse notes that the client's lochia has saturated one perineal pad. In light of the knowledge of expected lochial flow, what should the nurse conclude that this indicates?

Lochial flow within expected limits

At 42 weeks' gestation a client gives birth to an 8-lb 5-oz newborn. On examining the infant, what does the nurse expect to observe? (Select all that apply.)

Long nails Wrinkled skin

A client starting her second trimester asks a nurse in the prenatal clinic whether she can safely take an over-the-counter (OTC) medicine now that she is past the first 3 months of pregnancy. The nurse explains why she should consult with her health care provider before taking any oral medications. What physiological alteration associated with pregnancy may change the client's response to medication?

Longer gastrointestinal emptying time Gastrointestinal motility is reduced during pregnancy because of the high level of placental progesterone and displacement of the stomach superiorly and the intestines laterally and posteriorly; absorption of some drugs, vitamins, and minerals may be increased because of their slow passage through the gastrointestinal tract.

A 28-year-old woman is scheduled to undergo a laparoscopic bilateral salpingo-oophorectomy. What does a nurse expect to be the client's priority concern?

Loss of childbearing potential

A nurse is teaching a prenatal class about smoking during pregnancy. What neonatal consequence of maternal smoking should the nurse include in the teaching?

Low birth weight

A nurse determines that the husband of a client in the early phase of labor understands the teaching from childbirth classes when he helps his wife use the breathing pattern of:

Slow-chest

Which characteristics should alert the nurse to conclude that a male newborn is a preterm infant? (Select all that apply.)

Small breast buds Wrinkled thin skin Pinnae that remain flat when folded

What are the indicators of nutritional risk in pregnancy in a client who is of normal weight? (Select all that apply.)

Smoker Twin gestation

A nurse is observing the newborn of a known opioid user for signs of withdrawal. What clinical manifestations does the nurse expect to identify? (Select all that apply.)

Sneezing Hyperactivity High-pitched cry

A pregnant client who has a history of cardiac disease asks how she can relieve her occasional heartburn. The nurse should instruct the client to avoid antacids containing:

Sodium

A nurse assesses a new mother who is breastfeeding. The client asks how to care for her nipples. What should the nurse recommend?

Spreading breast milk on the nipples after the feeding and allowing them to air dry

The transmission of which microorganism that causes maternal mastitis is minimized by frequent handwashing by nursing staff members?

Staphylococcus aureus

A newborn has an Apgar score of 3 at 1 minute after birth. What is the immediate nursing action in response to this Apgar score?

Start resuscitation

After an unexpected emergency cesarean birth the client tells the nurse, "I failed natural childbirth." Which postpartum phase of adjustment does this statement most closely typify?

Taking-in

Jaundice develops in a newborn 72 hours after birth. What should the nurse tell the parents is the probable cause of the jaundice?

The physiological destruction of fetal red blood cells After birth, fetal erythrocytes hemolyze, releasing into the circulation bilirubin , which the immature liver cannot metabolize as rapidly as it is produced, resulting in physiological jaundice.

A client at 39 weeks' gestation arrives in the birthing suite reporting that she is having regular contractions. A vaginal examination reveals that the presentation is a double-footling breech. The practitioner decides to proceed to a cesarean birth under regional anesthesia. What is an important intervention to help prevent postoperative maternal complications?

Maintaining adequate hydration

What should the nurse explain to a newly pregnant client with cardiac disease?

Maintenance dosages of cardiac medications will probably be increased.

What type of lochia should the nurse expect to observe on a client's pad on the fourth day after a vaginal delivery?

Moderate serosa

What type of lochia should the nurse expect to observe on a client's pad on the fourth day after a vaginal delivery?

Moderate serosa On the third to fourth day the uterine discharge becomes pink to brown;

A nurse is planning a childbirth education class about maternal psychological and physiological changes as pregnancy nears term. Which problems and concerns should the nurse include in the presentation? (Select all that apply.)

Nesting needs increase. Anxiety about childbirth increases. Gastrointestinal motility decreases.

A pregnant client's blood test reveals an increased alpha-fetoprotein (AFP) level. What condition does the nurse suspect that this result indicates?

Neural tube defect

A nurse is trying to determine whether a pregnant woman's membranes have ruptured. What findings support the conclusion that they have ruptured? (Select all that apply.)

Nitrazine paper turns blue on contact with the fluid. Microscopic examination of the fluid reveals ferning/

A health care provider determines that a fetus is in a breech presentation. For which complication should the nurse monitor the client?

Nonreassuring fetal signs, indicating prolapse of the cord

A health care provider determines that a fetus is in a breech presentation. For which complication should the nurse monitor the client?

Nonreassuring fetal signs, indicating prolapse of the cord The feet or buttocks are not effective in blocking the cervical opening, and the cord may slip through and become compressed.

A client at 42 weeks' gestation has a reactive nonstress test. The nurse determines that the client understands what she was taught about the results when she is overheard telling her husband that the test was:

Normal because of an increase in fetal heart rate (FHR) with fetal movement

A 28-year-old woman seeks advice about oral contraceptives from the nurse in her company health office. What should the nurse tell her if she is a smoker?

Oral contraceptives can cause thrombophlebitis.

A client at 39 weeks' gestation is admitted for induction of labor. Knowing that several medications are used to induce labor, a nurse identifies those that may be prescribed. (Select all that apply.)

Oxytocin (Pitocin) Misoprostol (Cytotec) Dinoprostone (Prepidil)

What actions are part of nursing care during the fourth stage of labor for the client with a fourth-degree laceration? (Select all that apply.)

Pain management with oral analgesics Assessment of the site every 15 minutes Application of an ice pack for 20-minute intervals

A nurse is assessing a client who is being admitted for surgical repair of a rectocele. What signs or symptoms does the nurse expect the client to report? (Select all that apply.)

Painful intercourse Bearing-down sensations

A sonogram performed on a client in the third trimester demonstrates a low-lying placenta. The nurse should teach the client that she is at risk for:

Painless vaginal bleeding

The nurse presents a program on breast self-examination. After a return demonstration the nurse concludes that she needs to review certain aspects of the teaching program. Which behavior by one of the students supports this conclusion?

Palpating each breast while in the sitting position

The practice of separating parents from their newborn immediately after birth and limiting their time with the infant during the first few days after delivery contradicts studies of:

Parent-child attachment

A mother is concerned that her newborn will be exposed to communicable diseases when she goes home. While teaching the mother ways to decrease the risk of Infection; what type of immunity should the nurse explain was transferred to her baby through the placenta?

Passive natural

A histogram (hysterosalpingography [HSG]) is performed to determine whether a client has a tubal obstruction. The nurse explains to the client that infertility caused by a defect in the tube is most often related to:

Past infection

What should the nurse evaluate before continuing the administration of IV magnesium sulfate therapy to a client with preeclampsia?

Patellar reflexes and urinary output

Methods of relieving back pain are explained during a childbirth class. What activities identified by the client permit the nurse to conclude that the teaching has been understood? (Select all that apply.)

Pelvic rocking Forward tilting Sacral pressure

A nurse is teaching a client to care for her episiotomy after discharge. What priority instruction should the nurse include?

Perform perineal care after toileting until healing occurs.

A nurse is caring for several new mothers in the birthing unit, all in the taking-in phase of the postpartum period. What information is most appropriate for these clients at this time?

Perineal care During the taking-in phase a woman is primarily concerned with being cared for and being cared about. Infant feeding is best taught during the taking-hold phase of postpartum adjustment. Infant hygiene is best taught during the taking-hold phase of postpartum adjustment. Family planning is not a primary concern during the immediate postpartum period.

Sitz baths are ordered for a client with an episiotomy during the postpartum period. A nurse encourages her to take the sitz baths because they aid the healing process by:

Promoting vasodilation

While auscultating the lungs of a client admitted with severe preeclampsia, the nurse identifies crackles. What inference does the nurse make when considering the presence of crackles in the lungs?

Pulmonary edema has developed.

In her 37th week of gestation, a client with type 1 diabetes has amniocentesis to determine fetal lung maturity. The lecithin/sphingomyelin ratio is 2:1, phosphatidylglycerol is present, and creatinine is 2 mg/dL. What conclusion should the nurse draw from this information?

The newborn should be free from respiratory problems.

A pregnant client is concerned that she may have been infected with HIV. What information should a nurse include when counseling this client about HIV testing? (Select all that apply.)

The risks of passing the virus to the fetus What positive or negative test results indicate The emotional, legal, and medical implications of test results

The nurse reads the history of a neonate admitted to the nursery and discovers that the infant's mother was listed as gravida 1 para 1 before the baby was born. How should the nurse use these data to gather more information?

To consider that someone recorded the gravida and para incorrectly

A client at 16 weeks' gestation is scheduled for a sonogram followed by amniocentesis. The nurse instructs the client to drink 8 oz of fluid and not void before the sonogram. The nurse explains that this is done:

To improve visualization of the fetus A full bladder puts uterus in the optimal position for imaging because it raises the uterus out of the pelvis.

The healthcare provider of a woman who had a mastectomy has prescribed a Reach to Recovery visit. What does the nurse identify as the primary reason for the referral?

To prevent social isolation The Reach to Recovery visit helps the client meet her need to remain within her social milieu and informs her about available community resources.

The mother of a neonate with Down syndrome visits the clinic 1 week after delivery. She explains to the nurse that she is having problems feeding her baby. What is the probable cause of these feeding difficulties?

Tongue thrust Tongue extrusion, a reflex response that occurs when the tip of the tongue is touched, is characteristic of infants with Down syndrome and interferes with feeding; this reflex disappears around 4 months of age.

In what disorder is stimulation of labor contraindicated?

Total placenta previa

In what disorder is stimulation of labor contraindicated?

Total placenta previa A total placenta previa requires a cesarean birth ; early intervention helps ensure a healthy neonate and mother.

After her baby's birth a client wishes to begin breastfeeding. How can the nurse assist the client at this time?

Touching the infant's cheek adjacent to the nipple to elicit the rooting reflex

▶ A nurse is teaching a client how to self-administer a medicated douche. In which direction should the nurse instruct the client to direct the douche nozzle?

Toward the sacrum

A vaginal examination reveals that a client in labor is dilated 7 cm. Soon afterward she becomes nauseated and has the hiccups, and bloody show increases. What phase of labor does the nurse determine the client is entering?

Transition

A vaginal examination reveals that a client in labor is dilated 7 cm. Soon afterward she becomes nauseated and has the hiccups, and bloody show increases. What phase of labor does the nurse determine the client is entering?

Transition The transition phase is the most difficult phase of labor . Characterized by restlessness, irritability, nausea, and increased bloody show, it continues from 8 to 10 cm of dilation.

A male born at 28 weeks' gestation weighs 2 lb 12 oz. What characteristic does the nurse expect to observe?

Transparent red skin

When discussing dietary needs during pregnancy, a client tells the nurse that milk causes her to be constipated at times. What should the nurse teach the client?

Treat constipation when it occurs and continue drinking milk.

What is the safest position for a woman in labor when the nurse notes a prolapsed cord?

Trendelenburg A position in which the mother's head is below the level of the hips helps decrease compression of the cord and therefore maintains the blood supply to the fetus.

A client is receiving an oxytocin (Pitocin) infusion for induction of labor. The uterine graph on the electronic monitor indicates no rest period between contractions, and this is confirmed on palpation. What should the nurse do first?

Turn the oxytocin infusion off

During labor a client who has been receiving epidural anesthesia has a sudden episode of severe nausea, and her skin becomes pale and clammy. What is the nurse's immediate reaction?

Turning the client on her side

During labor a client who has been receiving epidural anesthesia has a sudden episode of severe nausea, and her skin becomes pale and clammy. What is the nurse's immediate reaction?

Turning the client on her side Maternal hypotension is a common complication of epidural anesthesia during labor, and nausea is one of the first clues that it has occurred.

During labor a client who has been receiving epidural anesthesia has a sudden episode of severe nausea, and her skin becomes pale and clammy. What is the nurse's immediate reaction?

Turning the client on her side Maternal hypotension is a common complication of epidural anesthesia during labor, and nausea is one of the first clues that it has occurred. Turning the client on her side will keep the uterus from putting pressure on the inferior vena cava, which causes a decrease in blood flow.

A client with osteoporosis has been receiving dietary information from the nurse. Which food selection by the client indicates that the nurse's dietary instruction was effective?

Turnip greens

The nurse is interpreting the results of a nonstress test (NST) on a client at 41 weeks' gestation. Which result after 20 minutes is suggestive of fetal reactivity?

Two accelerations of 15 beats/min lasting 15 seconds

A client is admitted with a marginal placenta previa. What should the nurse have available?

Two units of typed and screened blood

A 24-year-old client complains to the nurse in the women's health clinic that her breasts become tender before her menstrual period. What should the nurse recommend that the client do 1 week before an expected menses?

Decrease caffeine intake The client is exhibiting one symptom of premenstrual syndrome (PMS); eliminating food and beverages containing caffeine can limit breast swelling.

A newborn whose mother has type 1 diabetes is receiving a continuous infusion of fluids with glucose. What should the nurse do when preparing to discontinue the IV?

Decrease the rate slowly

A preterm infant with respiratory distress syndrome (RDS) has blood drawn for an arterial blood gas analysis. What test result should the nurse anticipate for this infant?

Decreased blood pH

A preterm infant with respiratory distress syndrome (RDS) has blood drawn for an arterial blood gas analysis. What test result should the nurse anticipate for this infant?

Decreased blood pH In addition to increased Pco2, hypoxia from inadequate oxygen/carbon dioxide exchange leads to anaerobic metabolism with an accumulation of acid by-products; both lower blood pH.

A local anesthetic (pudendal block) is administered to a client as second-stage labor begins. For what side effect does the nurse monitor for the client?

Decreased blood pressure Mild reactions, including vertigo, dizziness, and hypotension, occur because of vasodilation resulting from direct action of these medications on the mother's pelvic blood vessels.

A woman is being seen in the prenatal clinic at 36 weeks' gestation. The nurse is reviewing signs and symptoms that should be reported to health care provider with the mother. Which signs and symptoms require further evaluation by the health care provider? (Select all that apply.)

Decreased urine output Contractions that are regular and 5 minutes apart

A client's temperature is 100.4° F 12 hours after a spontaneous vaginal birth. What does the nurse suspect is the cause of the increased temperature?

Dehydration

A client's temperature is 100.4° F 12 hours after a spontaneous vaginal birth. What does the nurse suspect is the cause of the increased temperature?

Dehydration A client's temperature may be elevated to 100.4° F during the first 24 hours after delivery because of dehydration resulting from the exertion and stress of labor.

A pregnant woman who was admitted to the high-risk maternity unit for severe hyperemesis gravidarum is receiving total parenteral nutrition (TPN). Intralipids are not being administered. For what potential complication should the nurse monitor the client?

Dehydration TPN is a hypertonic solution that pulls fluid from the interstitial compartment into the intravascular compartment, resulting in diuresis and dehydration.

The nurse explains to a woman in her 24th week of pregnancy that absorption of medications taken orally during pregnancy may be altered as a result of:

Delayed gastrointestinal emptying

An infant is born with a bilateral cleft palate. Plans are made to begin reconstruction immediately. What nursing intervention should be included to promote parent-infant attachment?

Demonstrating positive acceptance of the infant

A pregnant client at 30 weeks' gestation begins to experience contractions every 5 to 7 minutes. She is admitted with a diagnosis of preterm labor. Although the client is being given tocolytic therapy her cervix continues to dilate, and it is determined that a preterm birth is inevitable. Which medication does the nurse expect the health care provider to prescribe?

Dexamethasone Dexamethasone is a glucocorticoid that stimulates the production of fetal lung surfactants, which are needed for fetal lung maturity; administration is started 48 hours before the expected birth. Norgestrel is a contraceptive hormone; it is not used for preterm labor. Aminophylline is a bronchodilator; it is not used for preterm labor. Magnesium sulfate is used for tocolytic therapy and has been somewhat effective in delaying preterm labor. However, this client's labor is progressing and the birth is inevitable.

A nurse is counseling a client with type 1 diabetes who has requested contraceptive information. On which method of contraception should the nurse place the most emphasis?

Diaphragm

What is the safest and most reliable birth control method for the nurse to recommend to a client with type 1 diabetes?

Diaphragm with a spermicidal gel

What is a nurse's most important concern when caring for a client with a ruptured tubal pregnancy?

Diminished cardiac output

A client undergoes anterior and posterior surgical repair of a cystocele and rectocele and returns from the postanesthesia care unit with an indwelling catheter in place. What should the nurse tell the client about the primary reasons for the catheter? (Select all that apply.)

Discomfort is minimized. Retention of urine is prevented. Pressure on the suture line is relieved.

A client in labor is receiving an oxytocin (Pitocin) infusion. What should the nurse do first when repetitive late decelerations of the fetal heart rate are observed?

Discontinue the oxytocin infusion

A primigravida who is at 38 weeks' gestation is undergoing a nonstress test. The nurse determines that the baseline fetal heart rate is 130 to 140 beats/min. It rises to 160 on two occasions and 157 once during a 20-minute period. Each of the episodes in which the heart rate is increased lasts 20 seconds. What action should the nurse take?

Discontinuing the test because the pattern is reassuring

A client is found to have preeclampsia, and bedrest at home is prescribed. It is doubtful that this client will be able to comply because she has two preschool children. What should be included in the plan of care that may help the client follow the prescribed regimen?

Discuss why bedrest is necessary

Sonography of a primigravida who is at 15 weeks' gestation reveals a twin pregnancy. The nurse reviews with the client the risks of a multiple pregnancy that were explained by the health care provider. Which condition does the client identify that indicates the need for further instruction about complications associated with a multiple gestation?

Down syndrome Chromosomal anomalies are not associated with a multiple gestation; therefore the client needs further instruction.

While reviewing laboratory results of clients seen at a maternity clinic, the nurse notes that one client's maternal serum α-fetoprotein level is lower than is typical. The nurse recognizes that this may be associated with:

Down syndrome Chromosomal trisomies such as Down syndrome may be marked by a lower-than-typical level of α-fetoprotein. The other options listed typically cause α-fetoprotein increases.

A client at 35 weeks' gestation calls the prenatal clinic, concerned that she has "not felt the baby move as much as usual." The most appropriate recommendation by the nurse is to have the client call the clinic with the results after she has:

Drunk a glass of orange juice and timed 10 fetal movements

The health care provider hands a neonate to a nurse immediately after birth. What should the nurse do next for the newborn?

Dry and provide skin-to-skin contact with the mother The priority is preventing heat loss; drying the newborn prevents heat loss through evaporation, and skin-to-skin contact with the mother provides a warm environment while promoting attachment.

A multigravida of Asian descent weighs 104 lb, having gained 14 pounds during the pregnancy. On her second postpartum day, the client's temperature is 99.2° F (37.3° C). She has had poor dietary intake since admission. What should the nurse do?

Encourage the family to bring in special foods preferred in their culture In family-centered childbearing, care should be adapted to the client's cultural system whenever possible.

What nursing intervention should be implemented routinely after a client has a vacuum aspiration abortion?

Encouraging the client to take the prescribed antibiotic medication

While caring for a client during labor, the nurse remembers that the second stage of labor:

Ends at the time of birth

Two days after birth a neonate's head circumference is 16 inches (40 cm) and the chest circumference is 13 inches (32.5 cm). What does the nurse infer from these measurements?

Enlarged head The enlarged head may indicate hydrocephalus. Average head circumference in the healthy newborn is 13.2 to 14 inches (33 to 35 cm), about 1 inch (2.5 cm) larger than the chest circumference.

A nurse in the fertility clinic is instructing a client who will be using progesterone gel vaginally in the treatment of luteal phase infertility. When discussing the side effects of progesterone, what should the nurse tell the client to expect?

Enlarged, tender breasts

The cervix of a client in labor is fully dilated and 100% effaced. The fetal head is at +3 station, the fetal heart rate ranges from 140 to 150 beats/min, and the contractions, lasting 60 seconds, are 2 minutes apart. What does the nurse expect to see when inspecting the perineum?

Enlarging area of caput with each contraction

During a prenatal interview at 20 weeks' gestation, the nurse determines that the client has a history of pica. What is the most appropriate nursing action?

Ensuring that the client's diet is nutritionally adequate

A client is taking a progesterone oral contraceptive (minipill). The nurse instructs the client to take one pill daily during the:

Entire menstrual cycle

A nurse discusses the type of anesthesia that will be used for a vaginal birth with a client who has class I cardiac disease. Which type of block is most appropriate for this client?

Epidural

At 30 weeks' gestation a client with class II cardiac disease expresses concern about her labor and asks the nurse what to expect. What does the nurse tell the client to expect if cardiac decompensation occurs?

Epidural anesthesia with a vacuum extraction birth Regional anesthesia does not compromise cardiovascular function as it provides pain relief. Vacuum extraction limits the mother's pushing during the second stage of labor, thereby limiting the workload of the heart and conserving energy. Major surgery is performed on clients with cardiac problems only when absolutely necessary because surgery adds additional stress to a compromised heart. Artificial rupture of the membranes may or may not be done; it is not specific to women with cardiac disease. Induced labor often is more stressful and painful than natural labor.

After a spontaneous vaginal delivery the client expresses concern because the newborn has a red rash with small papules on the face, chest, and back. What condition does the nurse recognize?

Erythema toxicum

Hydramnios is diagnosed in a primigravida at 35 weeks' gestation. What condition of the newborn is associated with hydramnios?

Esophageal atresia

Hydramnios is diagnosed in a primigravida at 35 weeks' gestation. What condition of the newborn is associated with hydramnios양수과다?

Esophageal atresia

The nurse is caring for a pregnant client who is undergoing an ultrasound examination during the first trimester. The nurse explains that an ultrasound during the first trimester is used to:

Estimate fetal age

A nurse is caring for a client with placenta previa who is in labor. What action is most important for the nurse to take?

Evaluating external blood loss by counting pads Evaluating external blood loss by counting pads will indicate whether bleeding is progressing toward maternal or fetal compromise.

Immediately after birth, a newborn is dried before being placed in skin-to-skin contact with the mother. What type of heat loss does this intervention prevent?

Evaporation

A pregnant client with sickle cell anemia visits the clinic each month for a routine examination. What additional observation should be made during every visit?

Evidence of pyelonephritis

A pregnant client with sickle cell anemia visits the clinic each month for a routine examination. What additional observation should be made during every visit?

Evidence of pyelonephritis Pregnant clients with sickle cell anemia are particularly vulnerable to infections , especially of the genitourinary tract; the examination of urine specimens should be performed frequently.

A client with a benign ovarian tumor undergoes laparoscopic surgery. What should the nurse include in the postoperative teaching?

Expect shoulder pain for 12 to 24 hours.

A 47-year-old client comes to the clinic for a Papanicolaou (Pap) smear. She tells the nurse that she has been experiencing hot flashes and that her periods have been occurring at longer, less regular intervals, with a scanty flow. What does the nurse conclude is the most likely cause of these changes?

Expected menopausal changes

What should the nurse's initial discussion include to best help new parents understand the unique characteristics of a newborn?

Expected movements and behaviors

What should the nurse's initial discussion include to best help new parents understand the unique characteristics of a newborn?

Expected movements and behaviors Information on typical behaviors helps parents understand the unique features of their newborn and promotes interaction and appropriate care.

When discussing future health management with a client who has had a total hysterectomy, the nurse advises regular physical examinations. The client agrees and adds, "It won't be so hard to go now that I won't need the pelvic examination and Pap smear." How should the nurse respond?

Explain why regular pelvic examinations and Pap smears of vaginal secretions will be necessary in the future.

A client is concerned about gaining weight during pregnancy. What should the nurse tell the client is the cause of the greatest weight gain during pregnancy?

Fetal growth

A nurse is observing the electronic fetal monitor as a client in labor enters the second stage. The nurse identifies early decelerations of the fetal heart rate with a return to the baseline at the end of each contraction. What does this usually indicate?

Fetal head compression

A client at 32 weeks' gestation is admitted in active labor. Her cervix is effaced and dilated 4 cm. Intramuscular betamethasone (Celestone) 12 mg is prescribed. What should the nurse tell the client about why the medication is being given?

Fetal lung maturity is accelerated. A steroid such as betamethasone (Celestone) or dexamethasone (Decadron) administered to the mother crosses the placenta and promotes lung maturity in the fetus.

A client with type 1 diabetes is scheduled for an amniocentesis at 36 weeks' gestation. She asks the nurse why this is being done so late in her pregnancy. What should the nurse consider before responding?

Fetal lung maturity may be evaluated.

A client with type 1 diabetes is scheduled for an amniocentesis at 36 weeks' gestation. She asks the nurse why this is being done so late in her pregnancy. What should the nurse consider before responding?

Fetal lung maturity may be evaluated. A test of the amniotic fluid can be used to determine the fetus's lung maturity; this determination can assist with the timing of a scheduled birth.

The nurse explains to a pregnant client undergoing a nonstress test that the test is a way of evaluating the condition of the fetus by comparing the fetal heart rate with:

Fetal movement

A pregnant client who is scheduled for a nonstress test (NST) asks a nurse how the test can show that "my baby is all right." The nurse explains that it is a way of evaluating the condition of the fetus by comparing the fetal heart rate (FHR) with:

Fetal physical activity

A nonstress test is scheduled for a client with preeclampsia. During the nonstress test the nurse concludes that if nonperiodic accelerations of the fetal heart rate occur with fetal movement, this probably indicates:

Fetal well-being Nonperiodic accelerations with fetal movement indicate fetal well-being. Early decelerations are associated with fetal head compression. Late decelerations are associated with uteroplacental insufficiency. Variable decelerations are associated with cord compression.

An infant born with hydrocephalus is to be discharged after insertion of a ventriculoperitoneal shunt. Which common complication should the nurse instruct the parents to report if it occurs at home?

Fever accompanied by decreased responsiveness

What is the primary outcome for client care in the third stage of labor?

Firmly contracted uterine fundus

During the examination of a client in labor, the cervix is determined to be dilated 4 cm. What stage of labor does the nurse record?

First The first stage of labor is from zero cervical dilation to full cervical dilation (10 cm). The second stage is from full cervical dilation to delivery. The prodromal stage is before cervical dilation begins. The transitional phase is first stage of labor, from 8 cm of dilation to 10 cm of dilation.

An infant is born in the breech position and diagnosed with Erb palsy (Erb-Duchenne paralysis). What clinical manifestation supports this conclusion?

Flaccid arm with the elbow extended on the affected side

A nurse is observing a newborn of 33 weeks' gestation. Which sign alerts the nurse to notify the health care provider?

Flaring nares

A nurse evaluates that a client who is taking oral contraceptives understands the related dietary teaching when the client states, "While I'm taking birth control pills I should increase my intake of foods containing:

Folic acid." Oral contraceptives are thought to cause deficiencies of folic acid, vitamin C, vitamin B6 and vitamin B12

A nurse evaluates that a client who is taking oral contraceptives understands the related dietary teaching when the client states, "While I'm taking birth control pills I should increase my intake of foods containing:

Folic acid." Oral contraceptives are thought to cause deficiencies of folic acid, vitamin C, vitamin B6 and vitamin B12.

A nurse is assessing a newborn with suspected retention of a fetal structure that will result in a congenital heart defect. Which fetal structures should undergo change after birth? (Select all that apply.)

Foramen ovale Ductus arteriosus

The clinic nurse is providing home care instructions for a client with pelvic inflammatory disease. What resting position should be recommended by the nurse?

Fowler The Fowler position facilitates localization of the infection by pooling exudate in the lower pelvis

A client is admitted to the birthing unit with uterine tenderness and minimal dark-red vaginal bleeding. She has a marginal abruptio placentae. The priority evaluation includes fetal status, vital signs, skin color, and urine output. What additional information is essential?

Fundal height

A nurse is caring for a group of postpartum clients. Which client is at the highest risk for disseminated intravascular coagulation (DIC)?

Gravida I who has had an intrauterine fetal death

A nurse is caring for a group of postpartum clients. Which client is at the highest risk for disseminated intravascular coagulation (DIC)?

Gravida I who has had an intrauterine fetal death Intrauterine fetal death is one of the risk factors for DIC; other risk factors include abruptio placentae, amniotic fluid embolism, sepsis, and liver disease.

During a client's labor the fetal monitor reveals a fetal heart pattern that signifies uteroplacental insufficiency. What is the nurse's first intervention?

Helping the client turn to the side-lying position

During the fourth stage of labor, the assessment of a primipara who has had a vaginal birth reveals a moderate to large amount of lochia rubra, a firm fundus that is at the umbilicus and deviated to the right, and pain that she rates as a 3 on a scale of 1 to 10. What is the priority nursing action?

Helping the client void A fundus that is deviated to the right during the fourth stage of labor commonly is caused by a distended bladder ; if the bladder remains distended, involution will be inhibited, resulting in a boggy uterus that is prone to hemorrhage.

A woman's pregnancy has been uneventful, and she has gained 25 lb. At term her hemoglobin level is 10.6 g/dL and her hematocrit is 31%. What does the nurse identify as the reason for these hemoglobin and hematocrit levels?

Hemodilution The increase in circulating blood volume during pregnancy is reflected in lower hemoglobin and hematocrit readings (physiological anemia of pregnancy).

A nurse is caring for a client who is receiving internal radiation for cancer of the cervix. For which adverse reactions to the radiotherapy should the client be monitored? (Select all that apply.)

Hemorrhage Increased temperature

A nurse is caring for a client who is receiving internal radiation for cancer of the cervix. For which adverse reactions to the radiotherapy should the client be monitored? (Select all that apply.)

Hemorrhage Increased temperature Excessive sloughing of tissue may cause hemorrhage and is considered an adverse reaction. Infection, marked by an increase in temperature, may also develop from excessive sloughing of tissue.

A nurse is caring for a pregnant client with thrombophlebitis. Which anticoagulant medication may be prescribed? (Select all that apply.)

Heparin (Hep-Lock) Enoxaparin (Lovenox)

A client presents to the clinic with complaints of nausea and amenorrhea and reports that she obtained a positive result on a home pregnancy test. Which component of the history is most indicative of pregnancy?

Her urine immunoassay test is positive,

A client who is at 12 weeks' gestation tells a nurse at the prenatal clinic that she is experiencing severe nausea and frequent vomiting. The nurse suspects that the client has hyperemesis gravidarum임신입덧 . What factor is frequently associated with this disorder?

High level of chorionic gonadotropin

A health care provider prescribes carboprost (Hemabate) to be administered to a postpartum client with intractable vaginal bleeding. What client factor should alert the nurse to question the prescription?

History of asthma Carboprost (Hemabate) is contraindicated in clients with asthma because of its respiratory side effects of coughing and dyspnea.

A client in preterm labor is receiving subcutaneous terbutaline (Brethine) tocolytic therapy. The nursing action that is most important during the initial administration of this medication is assessing the client's:

Pulse rate continuously Tachycardia is an expected side effect of terbutaline , a betamimetic agent. The pulse rate should be monitored continuously, and the rate should be no more than 120 beats/min.

After treatment for a bladder Infection; a client asks whether there is anything she can do to prevent cystitis in the future. What is the best response by the nurse?

Increase daily fluid consumption.

A preterm infant is receiving oxygen from an overhead hood. What nursing care is required while the infant is under the hood?

Putting a hat on the infant's head Oxygen has a cooling effect, and the infant should be kept warm so that metabolic activity and oxygen demands are not increased.

On a routine prenatal visit the sign or symptom that a healthy primigravida at 20 weeks' gestation will most likely report for the first time is:

Quickening

When performing Leopold maneuvers on a client who has been admitted to the birthing room, the nurse identifies a firm, round prominence over the symphysis pubis; a smooth, convex structure along her right side; irregular lumps along her left side; and a soft roundness in the fundus. What is the fetal position?

ROA

A 26-year-old primigravida experiencing severe abdominal pain is brought to the emergency department by ambulance with a suspected ruptured tubal pregnancy. What should the nurse do first?

Insert an intravenous catheter

A 26-year-old primigravida experiencing severe abdominal pain is brought to the emergency department by ambulance with a suspected ruptured tubal pregnancy. What should the nurse do first?

Insert an intravenous catheter The client is at risk for hypovolemic shock resulting from hemorrhage; administration of intravenous fluids is the priority.

A woman in the family planning clinic has decided to use the diaphragm for contraception. What should the nurse teach her about using a diaphragm?

Insert the diaphragm before intercourse and leave it in at least 6 hours after intercourse to kill all the sperm.

A couple, married for 5 years, want to start a family. When talking with the nurse the husband says, "Well, I guess we're going to have to jump into bed three or four times a day, every day, until it works." What is the nurse's best response?

Instructing them in the frequency and timing of intercourse to promote conception

A client at term is admitted in active labor. She has tested positive for HIV. Which intervention in the standard orders should the nurse question as a risk to the fetus?

Internal fetal scalp electrode

A client at term is admitted in active labor. She has tested positive for HIV. Which intervention in the standard orders should the nurse question as a risk to the fetus?

Internal fetal scalp electrode The electrode used for internal fetal monitoring pierces the fetal scalp; fetal exposure to maternal blood increases the risk of the fetus' contracting HIV.

A client with chronic hypertension and superimposed preeclampsia gives birth, at 39 weeks' gestation, to a 4-lb 12-oz infant. Which condition does this information suggest?

Intrauterine growth restriction

A client at 36 weeks' gestation exhibits oligohydramnios. What newborn complication should the nurse anticipate?

Intrauterine growth restriction (IUGR)

What is a common problem that affects the client in labor when an external fetal monitor has been applied to her abdomen?

Intrusion on movement

On her first visit to the prenatal clinic a client with rheumatic heart disease asks the nurse whether she has special nutritional needs. What supplements in addition to the regular pregnancy diet and prenatal vitamin and minerals will she need? (Select all that apply.)

Iron Folic acid

On her first visit to the prenatal clinic a client with rheumatic heart disease asks the nurse whether she has special nutritional needs. What supplements in addition to the regular pregnancy diet and prenatal vitamin and minerals will she need? (Select all that apply.)

Iron Folic acid Because pregnant women with heart disease are more likely to have anemia, there may be an additional need for iron and also for folic acid. If the pregnant client with heart disease is eating the recommended pregnancy diet and taking prenatal vitamin and mineral supplements, there is no additional need for calcium, vitamin C, and vitamin B12 .

A nurse is observing a newborn's respiratory rate. What clinical findings indicate that the rate is within the expected range?

Irregular, abdominal, 30 to 60/min The expected breathing pattern is abdominal and irregular in rhythm and depth (alternating between shallow and deep); the expected rate ranges from 30 to 60 breaths/min.Newborns' respirations are irregular and abdominal.

What signs and symptoms of withdrawal does the nurse identify in a postpartum client with a history of opioid abuse?

Irritability and muscle tremors

At a client's first prenatal visit, the healthcare provider performs a pelvic examination, stating that the client's cervix is bluish purple, which is known as the Chadwick sign. The client becomes concerned and asks whether something is wrong. The best response is "This is expected; it:

Is caused by increased blood flow to the uterus during pregnancy"

A pregnant client with severe preeclampsia is receiving an infusion of magnesium sulfate. What does the nurse identify as the main reason that this medication is administered?

It acts as an anticonvulsant.

A nurse in the newborn nursery receives a call from the emergency department saying that a woman with active herpes virus lesions gave birth in a taxicab while coming to the hospital. What does the nurse consider about the transmission of the herpes virus?

It can be acquired during a vaginal birth.

While showing a new mother how to care for her infant's umbilical cord stump, the nurse explains that the stump is a potential source of infection because:

It contains exposed tissue and blood

A client in the 38th week of gestation exhibits a slight increase in blood pressure. The health care provider advises her to remain in bed at home in a side-lying position. The client asks why this is important. What is the nurse's response regarding the advantage of this position?

It increases blood flow to the fetus.

Which statement is true regarding caput succedaneum in newborns?

It is swelling consisting of serum, blood, or both.

A newborn who has remained in the hospital because the mother had a cesarean birth is to be tested for phenylketonuria (PKU) on the morning of discharge. What should the nurse explain to the mother about the purpose of PKU testing?

It is used to measure protein metabolism.

A nurse notes that a client is voiding frequently in small amounts 8 hours after giving birth. What should the nurse conclude about this small output of urine during the early postpartum period?

It may indicate retention of urine with overflow.

A nurse is assessing a newborn for signs of hyperbilirubinemia (pathological jaundice). What clinical finding confirms this complication?

Jaundice that develops in the first 12 to 24 hours

What nursing care is most important for a newborn with respiratory distress syndrome (RDS)?

Keeping the infant in a warm environment

A newborn who is born at 36 weeks' gestation weighs 8 lb 13 oz (3997 g). How should the nurse document this finding?

LGA and preterm

A nurse performs Leopold's maneuvers on a newly admitted client in labor. Palpation reveals a soft, firm mass in the fundus; a firm, smooth mass on the mother's left side; several knobs and protrusions on the mother's right side; and a hard, round, movable mass in the pubic area with the brow on the right. On the basis of these findings, the nurse determines that the fetal position is:

LOA

A parent of a preterm infant in the neonatal intensive care unit, asks a nurse why the baby is in a bed with a radiant warmer. The nurse explains that preterm infants are at increased risk for hypothermia because they:

Lack the subcutaneous fat that usually provides insulation

What does the nurse expect the size of a newborn to be if the mother had inadequately controlled type 1 diabetes during her pregnancy?

Large for gestational age, near term

A woman is admitted for a hysterectomy and bilateral salpingo-oophorectomy. The nurse reviews the client's gynecological history. What condition does the client have that causes the nurse to anticipate an abdominal, rather than a vaginal, hysterectomy?

Large uterine fibroids

A client is scheduled to have a contraction stress test (CST) to determine fetal well-being. Which type of fetal heart rate (FHR) decelerations constitutes a nonreassuring outcome?

Late

☆ A client is scheduled to have a contraction stress test (CST) to determine fetal well-being. Which type of fetal heart rate (FHR) decelerations constitutes a nonreassuring outcome?

Late The fetus with a borderline cardiac reserve will show hypoxia, evidenced by a decreased FHR with minimal stress, making the test positive.

What factor identified by the nurse in a client's history places the client at increased risk for breast cancer?

Late beginning of childbearing

☆ A client who is at risk for seizures as a result of severe preeclampsia is receiving an IV infusion of magnesium sulfate. What findings cause the nurse to determine that the client is showing signs of magnesium sulfate toxicity? (Select all that apply.)

Respirations of 10/min Loss of patellar reflexes

A client who is at risk for seizures as a result of severe preeclampsia is receiving an IV infusion of magnesium sulfate. What findings cause the nurse to determine that the client is showing signs of magnesium sulfate toxicity? (Select all that apply.)

Respirations of 10/min Loss of patellar reflexes A high level of magnesium sulfate may depress respirations; if respirations are fewer than 12 breaths/min, immediate treatment is warranted. Toxicity results in diminished flexes or an absence of them; hypertonic (hyperactive) reflexes are related to preeclampsia

What is the most important parameter for the nurse to monitor during the first 24 hours after the birth of an infant at 36 weeks' gestation?

Respiratory distress

In her 36th week of gestation, a client with type 1 diabetes has a 9-lb 10-oz infant in a cesarean birth. For which condition should the nurse monitor this infant of a diabetic mother?

Respiratory distress syndrome

A nurse discusses herpes genitalis as part of a high school sex education program. The nurse explains to the students that herpes genitalis is:

Responsible for local as well as systemic reactions

What are the primary nursing interventions when a client is receiving an infusion of magnesium sulfate for severe preeclampsia? (Select all that apply.)

Restricting visitors Maintaining a quiet environment

A laboring client has asked the nurse help her to use a nonpharmacological strategy for pain management. Name the sensory simulation strategy.

Selecting a focal point and beginning breathing techniques

A nurse is being oriented to a prenatal clinic after graduation. The new nurse takes a course on several tests during pregnancy. Place the tests in the order in which they should be performed during pregnancy.

Sickle cell screening α-Fetoprotein (AFP) testing for neural tube defects Serum glucose for gestational diabetes Fetal movement test Group B Streptococcus culture

A client with mild preeclampsia is being treated on an outpatient basis. Three days of bedrest is prescribed. What position should the nurse encourage the client to maintain while in bed?

Side-lying

A nurse is caring for a client in active labor. What positions should the nurse encourage the client to assume to help promote comfort during back labor? (Select all that apply.)

Sitting Lateral Knee-chest

Which finding indicates the development of a complication resulting from the presence of bilateral cephalhematomas?

Skin color Cephalhematomas are gradually absorbed. As the hematoma resolves, hemolysis of red blood cells occurs, and jaundice may result

What should a nurse anticipate about the insulin requirements of a client with diabetes on her first postpartum day?

A sharp, sudden decrease

A nurse on the postpartum unit is assessing several clients. Which clinical finding requires immediate investigation?

A slow trickle of blood from the vagina

What should the nurse tell a new mother will be delayed until her newborn is 36 to 48 hours old?

Screening for phenylketonuria

A nurse is caring for a client who has severe preeclampsia. For which characteristic of eclampsia should the nurse monitor the client?

Seizures

A nurse is caring for a client in the first stage of labor and an external fetal heart monitor is in place. What do the tracings indicate?

Variable decelerations

A nurse on the postpartum unit is assessing several clients. Which clinical finding requires immediate investigation?

A slow trickle 조금씩 흐르는 소량의 액체of blood from the vagina

A premenopausal client is scheduled for a total abdominal hysterectomy. Which statement indicates to the nurse that the preoperative teaching has been understood?

"I won't have any more menstrual periods after this surgery."

After 18 months of unsuccessful attempts at conception by a client, primary infertility related to anovulatory cycles is diagnosed. Clomiphene citrate (Clomid) is prescribed. The nurse concludes that the client understands the teaching about the correct time to take the clomiphene when the she states:

"I'll start the pills on the fifth day of my cycle." The objective is to stimulate ovulation near the 14th day of the menstrual cycle, and this is achieved by taking the medication on the fifth through the ninth days;

After 18 months of unsuccessful attempts at conception by a client, primary infertility related to anovulatory cycles is diagnosed. Clomiphene citrate (Clomid) is prescribed. The nurse concludes that the client understands the teaching about the correct time to take the clomiphene when the she states:

"I'll start the pills on the fifth day of my cycle." he objective is to stimulate ovulation near the 14th day of the menstrual cycle, and this is achieved by taking the medication on the fifth through the ninth days; there is an increase in two pituitary gonadotropins luteinizing hormone and follicle-stimulating hormone, with subsequent ovarian stimulation. On the third day after the cycle there are insufficient hormones for clomiphene to be effective. The 16th day of the cycle is also too late for clomiphene to be effective.

The nurse is conducting teaching for a client being discharged after an abdominal hysterectomy. Which statement by the client indicates a need for further teaching?

"I'm glad I'll be able to get back into my jogging routine next week."

A nurse is instructing a client to cough and deep-breathe after an emergency cesarean birth. The client says, "Get out of here. Can't you see that I'm in pain?" Which response will be the most effective?

"If you can't cough, try taking six very deep breaths."

While inspecting her newborn a mother asks the nurse whether her baby has flat feet. How should the nurse respond?

"Infants' feet appear flat because the arch is covered with a fat pad."

A client is admitted with a diagnosis of torsion of the testes. How should the nurse respond when the client asks, "Why do I have to have surgery right now?"?

"Irreversible damage occurs after a few hours."

A client seeking advice about contraception asks a nurse about how an intrauterine device (IUD) prevents pregnancy. How should the nurse respond?

"It produces a spermicidal intrauterine environment."

An almost term client reports that her fetus is moving less this week than last week. Which responses are appropriate? (Select all that apply.)

"It would be good for you to come to labor and delivery to be evaluated today." "Always call the health care provider if you're worried that your baby isn't moving enough." "Let me teach you how to conduct a kick count, and then you can call me when you've done one."

A client with mild preeclampsia is told that she must remain on bedrest at home. The client starts to cry and tells the nurse that she has two small children at home who need her. How should the nurse respond?

"Let's explore your available current support and opportunities for child care."

A married couple has been using oral contraceptives to delay pregnancy. When the wife misses her regular menstrual period, she decides to find out whether she is pregnant. She tells the nurse that pregnancy may have occurred because she missed her contraceptive pills for 1 week when she had the flu. How should the nurse respond?

"You may be correct. The effect of contraceptive pills depends on their being taken on a regular schedule."

A client asks the nurse at the family planning clinic whether contraception is needed while she is breastfeeding. How should the nurse reply?

"You should use contraceptives, because ovulation may occur without a period."

A negative home pregnancy test may result if the woman performs the test:

10 days after intercourse took place

A nurse who is monitoring a newborn 3 minutes after birth remembers that the heart rate of a healthy, alert neonate may range between:

110 and 160 beats/min

A client who had a child with Tay-Sachs disease is pregnant and is to have an amniocentesis to determine whether the fetus has the disease. The nurse counsels her to plan for the procedure at the optimal time for the procedure at:

14 to 16 weeks' gestation

The nurse discusses the recommended weight gain during pregnancy with a newly pregnant client who is 5 feet 3 inches tall and weighs 125 lb. The nurse explains that with the recommended weight gain, at term the client should weigh about:

150 lb A weight of 150 lb would put the client within the recommended weight gain of at least 25 lb for a woman who was of average weight for her height before pregnancy.

A primigravida in her seventh week of gestation asks the nurse when she can expect to feel her baby move. The nurse replies that quickening usually occurs in the: 20th week

20th week Most primigravidas feel movement by the 20th week of gestation.

When a nurse brings a newborn to a mother, the mother comments about the milia on her infant's face. What information should the nurse include when responding?

Avoid squeezing them and don't try to wash them off.

A nurse teaches a woman who is planning to breastfeed how to relieve breast engorgement. The nurse determines that further teaching is necessary when the woman states that she will:

Breastfeed the infant less frequently

A 2-day-old infant who weighs 6 lb (2722 g) is fed formula every 4 hours. Newborns need about 73 mL of fluid per pound of body weight each day. In light of this information, approximately how much formula should the infant receive at each feeding? A 2-day-old infant who weighs 6 lb (2722 g) is fed formula every 4 hours. Newborns need about 73 mL of fluid per pound of body weight each day. In light of this information, approximately how much formula should the infant receive at each feeding?

2 to 3 oz Infants require about 73 mL of fluid per pound and 60 calories a day per pound for growth. The infant's weight of 6 lb × 73 mL of fluid = 438 mL. If fed every 4 hours the infant will have six feedings: 438 ÷ 6 = 73 mL; 73 ÷ 30 (30 mL/oz) = 2.4 oz. Therefore the infant should be offered 2 to 3 oz per feeding. One or 2 oz is inadequate for this newborn. Three to 5 oz is excessive for this newborn.

At 6 weeks' gestation a client is found to have gonorrhea. What medication does a nurse expect the health care provider to prescribe?

Ceftriaxone (Rocephin)

A client at 9 weeks' gestation asks the nurse in the prenatal clinic whether she may have chorionic villi sampling (CVS) performed during this visit. What should the nurse keep in mind as the optimal time for CVS while formulating a response?

At 10 weeks but no later than 12 weeks At 8 weeks but no later than 12 weeks is the ideal time for CVS; this gives the client time to consider other options if a problem is discovered.

A neonate weighing 5 lb 6 oz (2438 g) is born in a cesarean birth and admitted to the newborn nursery. What range of resting respiratory rate should the nurse anticipate?

30 to 60 breaths/min

A nurse is caring for several pregnant clients in the prenatal clinic. Which client causes the most concern because of her predisposition to placenta previa?

30 years old, gravida 6, para 5 Multiple past pregnancies tend to deplete the ability of the endometrial lining in the fundal portion of the uterus to support the placenta; the placenta then implants in the lower segment.

A 24-year-old client who has been told that she is pregnant is at her first prenatal visit. She is 5 feet 6 inches tall and weighs 130 lb. What should the nutrition plan regarding her daily caloric intake include?

340 more calories during the second trimester

A nurse is obtaining the health history of a woman who is visiting the prenatal clinic for the first time. She states that she is 5 months pregnant. For what positive sign of pregnancy should the nurse look in this patient?

Audible fetal heartbeat The presence of the fetal heartbeat is a positive sign of pregnancy.

A newborn of 30 weeks' gestation has a heart rate of 86 beats/min and slow, irregular respirations. The infant grimaces in response to suctioning, is cyanotic, and has flaccid muscle tone. What Apgar score should the nurse assign to this neonate?

A heart rate of less than 100 beats/min = 1; slow and irregular respirations = 1; grimaces in response to suctioning = 1; flaccid muscle tone = 0; and cyanosis = 0. This infant's Apgar score is 3. A score of 2 is too low. A score of 4 is too high, as is a score of 5.

Five minutes after being born, a newborn is pale; has irregular, slow respirations; has a heart rate of 120 beats/min; displays minimal flexion of the extremities; and has minimal reflex responses. What is this newborn's Apgar score?

5 According to the Apgar scoring system, the newborn receives 2 points for heart rate, 0 for color, 1 for respiratory effort, 1 for muscle tone, and 1 for reflex irritability. An Apgar score of 3 is low. Scores of 5 and 6 are higher, but the newborn may still require stimulation and oxygen.

The nurse instructs a pregnant client in the sources of protein that can be used to meet the increased daily requirement during pregnancy. How many grams of protein should the client eat each day?

60 g

A primigravida is admitted to the emergency department with a sharp, shooting pain in the lower abdomen and vaginal spotting. A ruptured tubal pregnancy is diagnosed. During what week of gestation does this condition most commonly occur?

6th

A nitrazine test strip that turns deep blue indicates that the fluid being tested has a pH of:

7.5

One minute after birth a nurse notes that a newborn is crying, has a heart rate of 140 beats/min, is acrocyanotic, resists the suction catheter, and keeps the arms extended. What Apgar score should the nurse assign to the newborn? Record your answer using a whole number. ___

8

One minute after birth a nurse assesses a newborn and auscultates a heart rate of 90 beats/min. The newborn has a strong, loud cry; moves all extremities well; and has acrocyanosis but is otherwise pink. What is this neonate's Apgar score?

8 A heart rate slower than 100 beats/min receives 1 point , and color (acrocyanosis—body pink, extremities blue) receives 1 point; the respiratory rate, muscle tone, and reflex irritability each get a score of 2, for a total of 8. A score of 9 is too high. An Apgar score of 7 is too low, as is a score of 6 is too low.

A nurse assesses a newborn 1 minute after birth. The body is pink with blue extremities; the heart rate is 122 beats/min; the legs are withdrawn when the soles are flicked, respiration is easy, with no evidence of distress; and the arms and legs are flexed and moving vigorously. What Apgar score should the nurse document in the newborn's medical record?

9

A 26-year-old woman whose sister recently had a mastectomy calls the local women's health center for an appointment for a mammogram. What should the nurse tell the client when preparing her for the mammogram?

A 26-year-old woman whose sister recently had a mastectomy calls the local women's health center for an appointment for a mammogram. What should the nurse tell the client when preparing her for the mammogram?

A client with severe preeclampsia in the high-risk unit is receiving an infusion of magnesium sulfate. If eclampsia were to occur, what action would the nurse take first?

A client with severe preeclampsia in the high-risk unit is receiving an infusion of magnesium sulfate. If eclampsia were to occur, what action would the nurse take first?

What should a nurse include in the teaching plan for a couple seeking information about family planning?

A condom must be held in place by the rim when the penis is withdrawn from the vagina.

A client at 32 weeks' gestation is admitted to the prenatal unit in preterm labor. An infusion of magnesium sulfate is started. What physiological response indicates to the nurse that the magnesium sulfate is having a therapeutic effect?

A decrease in frequency and duration of contractions

A nurse decides on a teaching plan for a new mother and her infant. What should the plan include?

A demonstration and explanation of infant care

A nurse prepares to administer vitamin K to a newborn. Why is vitamin K given specifically to newborns?

A newborn's intestinal tract does not synthesize it for several days after birth.

What type of respirations does the nurse expect to identify in a healthy newborn?

Abdominal and irregular A newborn's respirations are abdominal, diaphragmatic, and irregular; the rate varies from 30 to 60 breaths/min. Retractions are a sign of respiratory distress. A newborn's respirations are abdominal, not thoracic. Stertorous breathing may indicate respiratory distress .

Four days after a vaginal hysterectomy a client calls the follow-up service and tells the nurse that she has a yellowish-green vaginal discharge. The nurse advises the client to return to the clinic for an evaluation. Which symptoms are suggestive of a vaginal infection? (Select all that apply.)

Abdominal pain Rising temperature

A nurse is checking the external fetal monitor of a client in active labor. Which fetal heart pattern indicates cord compression?

Abrupt decreases in fetal heart rate that are unrelated to the contractions

A 16-year-old primigravida who appears to be at or close to term arrives at the emergency department stating that she is in labor and complaining of pain continuing between contractions. The nurse palpates the abdomen, which is firm and shows no sign of relaxation. What problem does the nurse conclude that the client is experiencing?

Abruptio placentae

A client is found to have gestational hypertension in the 22nd week of gestation. What is a major complication of hypertensive disease associated with pregnancy that the nurse should anticipate?

Abruptio placentae

A client is found to have gestational hypertension in the 22nd week of gestation. What is a major complication of hypertensive disease associated with pregnancy that the nurse should anticipate? Abruptio placentae

Abruptio placentae

A 16-year-old primigravida who appears to be at or close to term arrives at the emergency department stating that she is in labor and complaining of pain continuing between contractions. The nurse palpates the abdomen, which is firm and shows no sign of relaxation. What problem does the nurse conclude that the client is experiencing?

Abruptio placentae Abruptio placentae indicates premature placental separation; the classic signs are abdominal rigidity, a tetanic uterus, and dark-red bleeding

A 16-year-old primigravida who appears to be at or close to term arrives at the emergency department stating that she is in labor and complaining of pain continuing between contractions. The nurse palpates the abdomen, which is firm and shows no sign of relaxation. What problem does the nurse conclude that the client is experiencing?

Abruptio placentae Abruptio placentae indicates premature placental separation; the classic signs are abdominal rigidity, a tetanic uterus, and dark-red bleeding.

A pregnant woman who is in the third trimester arrives in the emergency department with vaginal bleeding. She states that she snorted cocaine approximately 2 hours ago. Which complication does the nurse suspect as the cause of the bleeding?

Abruptio placentae Abruptio placentae is associated with cocaine use; it occurs in the third trimester.

A nurse is caring for a client with severe preeclampsia who is receiving magnesium sulfate. What side effects indicate that the serum magnesium level may be excessive? (Select all that apply.)

Absence of the knee-jerk reflex Respiratory rate of 11 breaths/min

A 16-year-old girl who has become sexually active asks the nurse, "What's the most effective way to prevent a pregnancy?" Which method of preventing pregnancy should the nurse tell her is most effective?

Abstaining from sexual intercourse

Although a client in labor is prepared and plans to participate in the labor and birth process, she states that she is in severe discomfort. The nurse administers the prescribed butorphanol (Stadol). Which phase of labor is the safest time for the nurse to administer this medication?

Active phase

A nurse takes into consideration that the effect PKU has on the infant's development will depend on:

Adherence to a corrective diet instituted early

A nurse takes into consideration that the effect PKU has on the infant's development will depend on:

Adherence to a corrective diet instituted early In phenylketonuria (PKU), adherence to a specific diet is necessary for optimal physical growth with little or no adverse effects on mental development; a diet that is instituted late will not reverse brain damage.

The clinic nurse is planning care for a client found to have chlamydia. Which treatment should the nurse plan to implement?

Administration of azithromycin (Zithromax) 1 g orally in a single dose

A couple in their late 30s, expecting their first child, plans to have an amniocentesis. At what point in the pregnancy should the nurse tell the couple that the test it will be scheduled?

After the 14th week of pregnancy

The nurse teaches a high school sex education class that herpes genitalis infection cannot be cured but that the disease is marked by remissions and exacerbations. What else should the students be taught about this infection?

Although exacerbations occur they are not as severe as the initial episode.

The nurse is caring for a first-time mother at her first prenatal visit. The client confides, "I'm not sure about all this." Which research-based knowledge guides a nurse regarding the emotional factors of pregnancy?

Ambivalence and anxiety about mothering are common.

A client in her 10th week of pregnancy exhibits presumptive signs of pregnancy that the nurse may detect. (Select all that apply.)

Amenorrhea Breast changes Urinary frequency

A nurse provides a list of foods for a breastfeeding client with phenylketonuria (PKU) to avoid. Which nutrient is included on the list?

Amino acids PKU is an inborn error of metabolism involving an inability to metabolize phenylalanine, an essential amino acid. Lactose, glucose, and fatty acids are all metabolized by people with PKU.

While monitoring the fetal heart rate (FHR) of a client in labor, the nurse identifies an increase of 15 beats more than the baseline rate of 135 beats/min that lasts 15 seconds. How should the nurse document this event?

An acceleration

While monitoring the fetal heart rate (FHR) of a client in labor, the nurse identifies an increase of 15 beats more than the baseline rate of 135 beats/min that lasts 15 seconds. How should the nurse document this event?

An acceleration An acceleration is an abrupt increase in FHR above the baseline of 15 beats/min for 15 seconds; if the acceleration persists for more than 10 minutes, it is considered a change in baseline rate.

A nurse is performing the Ortolani test on a newborn. Which finding indicates a positive result?

An audible click on abduction

A client who expected to use the Lamaze technique throughout labor has an emergency cesarean birth. Three days later the client is found crying and tells the nurse that she is extremely disappointed because a cesarean birth was necessary. She asks the nurse why this happened to her. On what factor should the nurse base a response?

An emergency cesarean birth is traumatic psychologically because of the loss of the expected birth experience.

On a return visit to the fertility clinic a couple requests fertility drugs because, despite having a 28-day menstrual cycle and temperature readings that demonstrate an ovulatory pattern, the woman has been unable to conceive. What should the nurse explain to the couple?

An examination of semen will be needed.

Twenty-four hours after an uncomplicated labor and birth a client's complete blood count reveals a white blood cell (WBC) count of 17,000/mm3. How should the nurse interpret this WBC count?

An expected response to the process of labor and birth

After an abdominal hysterectomy the client returns to the unit with an indwelling catheter. The nurse notes that the urine in the client's collection bag has become increasingly sanguineous. What complication does the nurse suspect?

An incisional nick in the bladder

Respiratory acidosis is confirmed in a neonate with respiratory distress syndrome when the laboratory report reveals:

An increased Paco 2 of 55 mm Hg

A female client came to the clinic with suspected primary syphilis. What sign of primary syphilis does the nurse expect the client to exhibit?

An indurated painless nodule on the vulva that is draining This is the description of a chancre, which is the initial sign of syphilis. Flat wartlike plaques around the vagina and anus are condylomata, which are typical of the secondary stage of syphilis. Glistening patches in the mouth covered with a yellow exudate are typical of the secondary stage of systemic involvement, which occurs from 2 to 4 years after the disappearance of the chancre. A maculopapular rash on the palms and soles is typical of the secondary stage.

During a pelvic examination of a 24-year-old woman, the nurse suspects a vaginal infection because of the presence of a white curdlike vaginal discharge. What other assessment supports a fungal vaginal infection?

An itchy perineum

During the discharge conference with a client who has had a hysterectomy the nurse includes instructions for avoiding the thromboembolic phenomena that may occur as a complication. What should these instructions include?

Avoid sitting for long periods of time.

A nurse is admitting a pregnant client who has mitral valve stenosis to the high-risk unit. What prophylactic medication does the nurse anticipate administering during the intrapartum period?

Antibiotic Clients who have mitral valve stenosis are administered prophylactic antibiotic therapy to minimize the development of streptococcal infections that may cause endocarditis.

After a client gives birth, what physiological occurrence indicates to the nurse that the placenta is beginning to separate from the uterus and is ready to be expelled?

Appearance of a sudden gush of blood

What should a nurse teach a non-nursing mother to help relieve the discomfort of engorgement?

Apply cold packs to the breasts frequently.

A client who has just begun breastfeeding complains that her nipples feel very sore. What should the nurse encourage the mother to do? (Select all that apply.)

Apply cool packs to her breasts to reduce the discomfort Take the analgesic medication prescribed to limit the discomfort Assume a different position when breastfeeding to adjust the infant's sucking

A newborn male is being discharged 4 hours after having had a circumcision. What should the nurse instruct the mother to do?

Apply the diaper loosely for several days

One statement by a breastfeeding mother that indicates that the nurse's teaching about stimulating the let-down reflex has been successful is "I will:

Apply warm packs and massage my breasts before each feeding."

Take your temperature daily and notify the clinic if it goes above 99.6° F.

Apply warm packs and massage my breasts before each feeding."

A client in labor is experiencing discomfort because her fetus is in the occiput posterior position. What nursing action will help relieve this discomfort?

Applying pressure against her sacrum

An infant born at 40 weeks' gestation weighs 6 lb 13 oz (3090 g). What category describes this neonate?

Appropriate for gestational age (AGA) and term

A pregnant woman at 34 weeks' gestation is being seen at the clinic. The client's blood pressure is 166/100 mm Hg and her urine is +3 for protein. She states that she has a severe headache and occasional blurred vision. Her baseline blood pressure was 100/62 mm Hg. What is the priority nursing action?

Arranging transportation to the hospital

At her first prenatal clinic visit a primigravida has blood drawn for a rubella antibody screening test, and the results are positive. What intervention is important when the nurse discusses this finding with the client?

Asking her whether she has had German measles and when she had the disease The positive result indicates that the client has had rubella or was vaccinated. The nurse should determine whether she has had the disease, because it is important to know whether it was before or after she became pregnant; if she had rubella at the start of her pregnancy, the fetus is at risk.

A client's temperature may be elevated to 100.4° F during the first 24 hours after delivery because of dehydration resulting from the exertion and stress of labor.

Asking the client questions, using a postpartum depression scale

After reading that nutrition during pregnancy is important for optimal growth and development of a baby, a pregnant woman asks the nurse what foods she should be eating. The nurse begins the teaching/learning process by:

Asking the client what she usually eats at each meal

While caring for a client who gave birth 1day ago, the nurse determines that the client's uterine fundus is firm at one fingerbreath below the umbilicus, blood pressure is 110/70 mm Hg, pulse is 72 beats/min, and respirations are 16 breaths/min. The client's perineal pad is saturated with lochia rubra. What is the priority nursing action?

Asking the client when she last changed the perineal pad

While caring for a client who gave birth 1day ago, the nurse determines that the client's uterine fundus is firm at one fingerbreath below the umbilicus, blood pressure is 110/70 mm Hg, pulse is 72 beats/min, and respirations are 16 breaths/min. The client's perineal pad is saturated with lochia rubra. What is the priority nursing action?

Asking the client when she last changed the perineal pad The amount of lochia would be excessive if the pad were saturated in 15 minutes; saturating the pad in 2 hours is considered heavy bleeding. If the pad has not been changed for a longer period, this could account for the large quantity of lochia. These findings cannot be supported without additional information. Oxytocics are administered for uterine atony; the need for this is not supported by the assessment of a firm fundus. The vital signs do not indicate hemorrhage; further assessment is needed before the nurse comes to this conclusion.

The clinic nurse is reviewing the dietary intake of a 16-year-old who is 12 weeks pregnant. What is the nurse's next action?

Asking the client, "How many servings of dairy do you generally consume each day?"

A client who is to undergo dilation and curettage, and conization of the cervix for cancer appears tense and anxious. What is the best approach for the nurse to support the client emotionally?

Asking whether something is troubling the client and whether she'd like to talk about it

What is the best nursing action for a client in active labor whose cervix is dilated 4 cm and 100% effaced with the fetal head at 0 station?

Assist the client's coach in helping her with the use of breathing techniques

A client at 9 weeks' gestation asks the nurse in the prenatal clinic whether she may have chorionic villi sampling (CVS) performed during this visit. What should the nurse keep in mind as the optimal time for CVS while formulating a response?

At 10 weeks but no later than 12 weeks

A woman has made the decision to have breast augmentation surgery, and the procedure is to be performed on an outpatient basis. As part of the preoperative protocol, the nurse provides teaching regarding the discharge instructions. Which instructions apply to this type of surgery? (Select all that apply.)

Avoid taking aspirin or NSAIDs (e.g., ibuprofen [Advil]) for pain relief. Sleep with your head and torso elevated for at least 1 week. Take your temperature daily and notify the clinic if it goes above 99.6° F.

A client at 35 weeks' gestation is experiencing contractions. Her cervix is dilated 2 cm. The nurse teaches the client that sexual activity, particularly intercourse, should be:

Avoided to limit the onset of labor A client who is experiencing preterm contractions with the cervix dilated 2 cm should avoid coitus, regardless of position; prostaglandins in semen may stimulate labor, and penile contact with the cervix may increase myometrial contractility.

What characteristic does the nurse anticipate in an infant born at 32 weeks' gestation?

Barely visible areolae and nipples

What characteristic does the nurse anticipate in an infant born at 32 weeks' gestation?

Barely visible areolae and nipples Breast tissue is not palpable in a newborn of less than 33 weeks' gestation.

The fetus of a woman in labor is at +1 station. At what place in the pelvic area does the nurse conclude that the presenting part is located?

Below the ischial spines

After performing Leopold maneuvers on a laboring client, a nurse determines that the fetus is in the right occiput posterior (ROP) position. Where should the Doppler ultrasound transducer be placed to best auscultate fetal heart tones?

Below the umbilicus on the right side

A nurse is reviewing the laboratory report of a newborn whose hematocrit level is 45%. What value denotes a healthy infant?

Between 45% and 65% The expected hematocrit level for a healthy newborn is between 45% and 65%. Less than 40% is below the expected level and is considered anemia. More than 75% is high and is considered polycythemia. Between 65% and 75% is above the expected range.

A client seeking family planning information asks the nurse during which phase of the menstrual cycle an intrauterine device (IUD) should be inserted. Before responding the nurse recalls that the insertion usually is done:

Between the first and fourth days of the cycle

A client whose membranes have ruptured is admitted to the birthing unit. Her cervix is dilated 3 cm and 50% effaced. The amniotic fluid is clear and the fetal heart rate is stable. What does the nurse anticipate?

Birth of the fetus within a day

A pregnant woman tells a nurse in the prenatal clinic that she knows that folic acid is very important during pregnancy and that she is taking a prescribed supplement. She asks the nurse what foods contain folic acid (folate) so she may add them to her diet in its natural form. Which foods should the nurse recommend? (Select all that apply.)

Black and pinto beans Enriched bread and pasta Legumes contain large amounts of folate , as do enriched grain products.

A nurse determines that a newborn has a cephalhematoma. What did the nurse note?

Bleeding between the parietal bone and periosteum confined within the suture line

A nurse determines that a newborn has a cephalhematoma. What did the nurse note?

Bleeding between the parietal bone and periosteum confined within the suture line Cephalhematoma is a collection of blood localized between the periosteum and the bony cranium caused by the rupture of blood vessels during the birth process; it does not cross suture lines.

Which information is most important for a large-for-gestational-age (LGA) infant of a diabetic mother (IDM)?

Blood glucose level less than 40 mg/dL At birth, circulating maternal glucose is removed; however, the IDM still has a high level of insulin, and rebound hypoglycemia may develop.

A nurse withholds methylergonovine maleate (Methergine) from a postpartum client. What clinical finding supports the withholding of the medication?

Blood pressure of 160/90 mm Hg

A nurse withholds methylergonovine maleate (Methergine) from a postpartum client. What clinical finding supports the withholding of the medication?

Blood pressure of 160/90 mm Hg Methylergonovine maleate can cause hypertension and should not be given to a client with an increased blood pressure.

A nurse who is caring for a client in labor uses nitrazine paper to test the pH of the client's leaking vaginal fluid. What color will the nitrazine paper turn if the leakage is amniotic fluid?

Blue

A 16-year-old adolescent at 24 weeks' gestation visits the prenatal clinic for the first time. After the physical examination she tells the nurse, "I can't believe how big I am. Will I get much bigger?" What information about adolescent growth and development does the nurse need to know before responding?

Body image is very important to adolescents, so pregnant teenagers are concerned about body size.

What does a nurse expect to find when checking the vital signs of a client in the early postpartum period?

Bradycardia with no change in respirations

A nurse is caring for a preterm neonate with physiological jaundice who requires phototherapy. What is the action of this therapy?

Breaks down the bilirubin into a conjugated form

A client who is visiting the family planning clinic is prescribed an oral contraceptive. As part of teaching, the nurse plans to inform the client of the possibility of:

Breakthrough bleeding Breakthrough bleeding, or midcycle bleeding, commonly occurs when women start using oral contraceptives.

A nurse is reviewing a postmenopausal client's history, which reveals that the client previously received hormonal replacement therapy (HRT) as treatment for osteoporosis. For which problem does HRT increase the client's risk?

Breast cancer

A nurse is estimating a newborn's gestational age. What parameters should the nurse evaluate? (Select all that apply.)

Breast size Genital development

A few weeks after discharge, a postpartum client experiences mastitis and telephones for advice concerning breastfeeding. The nurse notifies the practitioner to have antibiotics prescribed. What should the nurse recommend that the client do?

Breastfeed often to keep the breasts empty.

A client in active labor becomes very uncomfortable and asks a nurse for pain medication. Nalbuphine (Nubain) is prescribed. How does this medication relieve pain?

By acting on opioid receptors to reduce pain Nalbuphine (Nubain) is classified as an opioid analgesic and is effective in relieving pain; it induces little or no newborn respiratory depression. Nalbuphine does not induce amnesia, act as an anesthetic, or induce sleep.

A client who had a cesarean birth is unable to void 3 hours after the removal of an indwelling catheter. How can the nurse evaluate whether the client's bladder is distended?

By palpating the client's suprapubic area gently

A 7-lb, 4-oz (3290-g) boy is admitted to the nursery and placed in a warm crib. The neonate begins to choke on mucus. How should the nurse suction him with a bulb syringe?

By suctioning the mouth before the nostrils

When a client is being given an intravenous infusion of magnesium sulfate, the nurse should have its antidote readily available. Which of the following medications would the nurse administer if toxicity were to occur?

Calcium gluconate Calcium gluconate will reverse the central nervous system depressant action of magnesium sulfate. Protamine sulfate is the antidote for heparin toxicity. Sodium bicarbonate counteracts acidosis. Naloxone hydrochloride is an opiate antagonist.

After a client's membranes rupture spontaneously, the nurse sees the umbilical cord protruding from the vagina. Place the nursing interventions in order of priority.

Call for assistance and don sterile gloves Insert two fingers into the vagina and exert upward pressure against the fetal presenting part Put a rolled towel under one hip and place in the modified Sims position Administer oxygen to the mother and monitor fetal heart tones

Which behavior indicates to a nurse that a new mother is in the taking-hold phase?

Calling the baby by name The mother has moved into the taking-hold phase when she takes control and becomes actively involved with her infant and calls the infant by name

A client has just been told that she has cervical erosion. The nurse would expect to help explain that early treatment of the erosion can help prevent:

Cancer of the cervix

The nurse visualizes and palpates a generalized, soft, edematous area of the scalp on the occiput of a newborn. What does the nurse suspect?

Caput succedaneum

The nurse visualizes and palpates a generalized, soft, edematous area of the scalp on the occiput of a newborn. What does the nurse suspect?

Caput succedaneum Pressure against the fetal head during labor can cause localized trauma, which results in edema that is not confined within the suture lines of the skull. The edematous area is usually generalized and on the occiput.

A nurse is teaching a prenatal class about the changes that occur during the second trimester of pregnancy. What cardiovascular changes should the nurse include? (Select all that apply.)

Cardiac output increases. Blood pressure decreases. The heart is displaced upward.

A nurse is caring for a client with tertiary syphilis. Which body system should the nurse monitor most closely?

Cardiovascular Syphilis is primarily a vascular disease; aortitis, valvular insufficiency, and aortic aneurysms are the most prevalent problems in tertiary syphilis.

A neonate is born with exstrophy of the bladder, and the parents are upset. They are told that corrective surgery will be performed as soon as possible. How can the nurse best help the parents at this time?

Caring for the newborn in the same manner as any other newborn

The nurse teaches a client about the increased need for vitamin A to meet the demands imposed by rapid fetal tissue growth during pregnancy. Which foods should the nurse encourage the client to ingest to meet this increased need? (Select all that apply.)

Carrots Sweet potatoes

A client with severe preeclampsia is receiving an IV infusion of magnesium sulfate. The nurse remembers that magnesium sulfate is a:

Central nervous system depressant that blocks neuromuscular transmissions. Eclamptic seizures may be prevented with the administration of IV magnesium sulfate, which is a central nervous system depressant. Although magnesium sulfate is a neuromuscular sedative that relaxes smooth muscle and decreases blood pressure, it is not considered an antihypertensive and is not given for that purpose. Magnesium sulfate decreases, not increases, the quantity of acetylcholine. Decreased uterine contractions are not associated with magnesium sulfate administration.

During the discharge examination of a 2-day-old newborn, the nurse observes an edematous area confined to the right side of the scalp. How should the nurse document this condition?

Cephalhematoma Cephalhematoma is a collection of blood beneath the periosteum of the skull bone; the blood mass does not cross the suture line and is confined to one side of the head. It is reabsorbed within 3 to 6 weeks.

How does the nurse know that a client at 40 weeks' gestation is experiencing true labor?

Cervical dilation

A primigravida at term is admitted to the birthing room in active labor. Later, when the client is dilated 8 cm, she tells the nurse that she has the urge to push. The nurse instructs her to pant-blow at this time because pushing can cause which of the following?

Cervical edema

A primigravida at term is admitted to the birthing room in active labor. Later, when the client is dilated 8 cm, she tells the nurse that she has the urge to push. The nurse instructs her to pant-blow at this time because pushing can cause which of the following?

Cervical edema The head cannot emerge when the cervix is not fully dilated. Pushing in this situation may cause cervical edema, predisposing the client to cervical laceration.

During prenatal classes the nurse teaches the difference between true labor and false labor. How does the nurse explain the difference?

Cervix effaces and dilates during true labor.

A client in labor is admitted to the birthing unit. Assessment reveals that the fetus is in a footling breech presentation. What should the nurse consider about breech presentations when caring for this client?

Cesarean birth probably will be necessary

A woman in labor with her third child is dilated 7 cm, and the fetal head is at station +1. The client's membranes rupture. What should the nurse do first?

Check the fetal heart rate while observing the color of the amniotic fluid

A woman in labor with her third child is dilated 7 cm, and the fetal head is at station +1. The client's membranes rupture. What should the nurse do first?

Check the fetal heart rate while observing the color of the amniotic fluid Fetal well-being is the priority. The fetal heart rate will reflect the fetus' response to the rupture of the membranes, and the color of the amniotic fluid will reveal whether there is meconium staining.

An estrogen-progestin oral contraceptive is prescribed for a client. Which adverse effects should the nurse teach the client to report to the health care provider? (Select all that apply.)

Chest pain Breast soreness Calf tenderness

A client who is in labor is admitted 30 hours after her membranes ruptured. For what condition does the nurse anticipate that the client is most at risk?

Chorioamnionitis The risk of developing chorioamnionitis (intra-amniotic infection) is increased with prolonged rupture of the membranes; foul-smelling fluid is a sign of infection. A prolapsed cord usually occurs shortly after the membranes rupture, not 1½ days later. Placenta previa is an abnormally implanted placenta; it is unrelated to ruptured membranes. Premature separation of the placenta is unrelated to ruptured membranes.

Which risk factors are associated with the future development of osteoporosis in women? (Select all that apply.)

Cigarette smoking Familial predisposition Inadequate intake of dietary calcium

A client has chosen not to have her son circumcised. What instruction should be included in discharge teaching for the care of an uncircumcised neonate?

Clean the penis with warm water at each diaper change.

In childbirth classes the nurse is teaching paced breathing techniques for use during labor. In which order should the breathing techniques be used as labor progresses?

Cleansing breaths Slow, deep breaths Modified paced breathing Pant-blow breathing Slow, exhalation pushing

A couple interested in family planning ask the nurse about the cervical mucus method of family planning. The nurse explains that with this method the couple must avoid intercourse when and a few days after the cervical mucus is:

Clear and stretchable

The day after a client has a cesarean birth, the indwelling catheter is removed. The nurse concludes that urinary function has returned when the:

Client voids 300 mL of urine within 4 hours of catheter removal

A neonate is tested for phenylketonuria (PKU) after formula feedings are initiated. The nurse explains to the parents that this is done to prevent:

Cognitive Impairment Screening for PKU facilitates early diagnosis and treatment, which can prevent cognitive impairment.

A nurse assesses a healthy 8-lb 8-oz (3860-gm) newborn who was given Apgar scores of 9 at 1 minute and 10 at 5 minutes. Which category of the Apgar score received a 1 rating at one minute?

Color Because of inadequate peripheral circulation at birth there is acrocyanosis (body pink, hands and feet blue), which merits 1 point for color . This is a common occurrence in a healthy newborn. The fetal heart rate ranges from 110 to 160 beats/min; a newborn heart rate of more than 100 beats/min is expected in a healthy newborn and merits 2 points. An adequate respiratory rate is evidenced by crying, which is expected in a healthy newborn and merits 2 points. Reflex irritability is represented by crying, which is expected in a healthy newborn and merits 2 points.

A nurse in the postpartum unit must complete several interventions before a client's discharge from the hospital. The nurse plans to delegate some of the tasks to the nursing assistant. Which activity must be performed by the nurse?

Comparing the identification bands of mother and infant

A nurse is teaching a pregnant client with sickle cell anemia about the importance of taking supplemental folic acid. Folic acid is important for this client because it:

Compensates for a rapid turnover of red blood cells

What should the nurse teach a client about performing breast self-examination?

Compress the nipples to check for discharge.

After a modified radical mastectomy a client has two portable wound drainage systems in place. What is an important intervention as the nurse cares for these drainage systems?

Compressing the drainage receptacles after emptying them to maintain suction

After a modified radical mastectomy a client has two portable wound drainage systems in place. What is an important intervention as the nurse cares for these drainage systems?

Compressing the drainage receptacles after emptying them to maintain suction Portable wound drainage systems are self-contained and may be emptied and compressed to reestablish negative pressure, which promotes drainage.

A client is being prepared for an emergency cesarean birth because of fetal compromise. What is the most important preoperative nursing action?

Confirming the signed consent

A client at 35 weeks' gestation who has had no prenatal care arrives in labor and delivery and is found to be 20 percent effaced and 2 cm dilated, with her membranes intact and contractions 3 minutes apart. The nurse notices some ruptured blisterlike vesicles in the genital area. What should the nurse's next action be?

Contacting the health care provider about the need for a cesarean birth

One hour after a birth a nurse palpates a client's fundus to determine whether involution is taking place. The fundus is firm, in the midline, and two fingerbreadths below the umbilicus. What should the nurse do next?

Continue periodic evaluations and record the findings Immediately after birth the uterus is 2 cm below the umbilicus; during the first several postpartum hours the uterus will rise slowly to just above the level of the umbilicus. These findings are expected, and they should be recorded.

A pregnant client tells the nurse that her husband is a chain smoker. What information should the nurse's teaching include?

Continued exposure to secondhand smoke is related to fetal growth restriction.

Three weeks after a client gives birth, a deep vein thrombophlebitis develops in her left leg and she is admitted to the hospital for bedrest and anticoagulant therapy. Which anticoagulant does the nurse expect to administer?

Continuous infusion of heparin Heparin is the medication of choice during the acute phase of a deep vein thrombosis; it prevents conversion of fibrinogen to fibrin and of prothrombin to thrombin.

A nurse is assessing a postpartum client for signs of an impending hemorrhage resulting from laceration of the cervix. Besides monitoring the client for a firm uterus, what other assessment is important?

Continuous trickling of blood

What potential complication should the nurse anticipate when a pregnant client has premature rupture of the membranes?

Cord prolapse Premature rupture of the membranes may permit prolapse of the umbilical cord if the fetal head is not engaged.

A 20-year-old developmentally disabled woman is a resident in a group home. She has had four abortions in the past 2 years, and the agency supervisor recommends that she be sterilized. It is obvious that the client is unable to exercise informed consent for sterilization. The nurse understands that the procedure cannot be performed without legal consent from the:

Court-appointed individual or group

A client at 43 weeks' gestation has just given birth to an infant with typical postmaturity characteristics. Which postmature signs does the nurse identify? (Select all that apply.)

Cracked and peeling skin Long scalp hair and fingernails Creases covering the neonate's full soles and palms

The nurse is caring for a newborn with caput succedaneum. The nurse is able to differentiate caput succedaneum from cephalhematoma because caput succedaneum features scalp edema that:

Crosses the suture line Scalp edema that crosses the suture line is the sign that differentiates between these two conditions; with caput succedaneum the swelling crosses the suture line, whereas it does not in cephalhematoma.

A nurse determines that a newborn is in respiratory distress. Which signs confirm respiratory distress in the newborn? (Select all that apply.)

Cyanosis Tachypnea Retractions

A nurse determines that a newborn is in respiratory distress. Which signs confirm respiratory distress in the newborn? (Select all that apply.)

Cyanosis Tachypnea Retractions Cyanosis occurs because of inadequate oxygenation. Tachypnea is a compensatory mechanism to increase oxygenation. Retractions occur in an effort to increase lung capacity. Crackles occur in the healthy newborn. Wheezing in the newborn is benign.

A nurse suspects that a newborn's mother had rubella during the first trimester of pregnancy. Which newborn problems support this assumption? (Select all that apply.)

Deafness Cardiac anomalies

A nurse suspects that a newborn's mother had rubella during the first trimester of pregnancy. Which newborn problems support this assumption? (Select all that apply.)

Deafness Cardiac anomalies Depending on the specific period of organogenesis when the mother contracted rubella , a variety of defects may occur. Deafness is a typical sign of a newborn affected by a mother who had rubella during early pregnancy. Cardiac anomalies are common in newborns if the mother had rubella during pregnancy during the time of organogenesis.

On reporting to the labor and delivery area a primipara indicates to the nurse that her contractions are occurring every 5 minutes. Upon further inquiry the nurse learns that the client has not attended any childbirth classes, and a cervical assessment reveals that she is in labor. When is the best time for the nurse to include education on simple breathing and relaxation techniques?

During the latent phase of the first stage of labor

On reporting to the labor and delivery area a primipara indicates to the nurse that her contractions are occurring every 5 minutes. Upon further inquiry the nurse learns that the client has not attended any childbirth classes, and a cervical assessment reveals that she is in labor. When is the best time for the nurse to include education on simple breathing and relaxation techniques?

During the latent phase of the first stage of labor During the latent phase of the first stage of labor the client is excited and open to learning.

The nurse discusses fetal weight gain with a pregnant client. When does it usually show a marked increase?

During the third trimester

A client at 36 hours' postpartum is being treated with subcutaneous enoxaparin (Lovenox) for deep vein thrombosis of the left calf. Which client adaptation is of most concern to the nurse who is monitoring the client?

Dyspnea

A client at 36 hours' postpartum is being treated with subcutaneous enoxaparin (Lovenox) for deep vein thrombosis of the left calf. Which client adaptation is of most concern to the nurse who is monitoring the client?

Dyspnea One complication of deep vein thrombosis is pulmonary embolism; dyspnea is a significant sign that should be reported immediately.

During a male newborn's first encounter with his mother the nurse encourages her to undress him. The mother strokes him with her whole hand and while looking at him intently says, "He feels so velvety, and he is going to be just as good looking as his daddy." The baby is alert and responsive while gazing at his mother. What is the nurse's assessment of this first mother-infant encounter?

Early parenting behavior

A client had a fourth-degree perineal laceration during the birth of her neonate. What should the nurse recommend to protect the area from additional trauma?

Eat a high-fiber diet with increased fluid intake.

A client is receiving antibiotics and antifungal medications for the treatment of a recurring vaginal infection. What should the nurse encourage the client to do to compensate for the effect of these medications?

Eat yogurt daily Yogurt contains Lactobacillus acidophilus, which replaces the intestinal flora destroyed by antibiotics. The other options are not relevant to antibiotics or intestinal flora.

A client who has missed two menstrual periods arrives at the prenatal clinic with vaginal bleeding and one-sided lower quadrant pain. What condition does the nurse suspect?

Ectopic pregnancy

What should the nurse emphasize in a class about childbirth?

Education; exercise, and breathing techniques

What recommendation should a nurse give to a client with fluid retention during pregnancy?

Elevate the lower extremities.

What should a nurse include in the discharge instructions for a woman who has undergone breast-conserving surgery (lumpectomy) for breast cancer?

Emphasizing the importance of breast self-examination

A nurse is planning care for a client who gave birth to a preterm male infant. What most common response does the nurse anticipate that the mother may experience?

Feelings of failure and loss of control

A primigravida is admitted with a ruptured fallopian tube resulting from a tubal pregnancy and surgery is performed to remove the fallopian tube. What should postoperative nursing care include?

Explaining that the client may still be capable of becoming pregnant Removing a fallopian tube does not impair the ovaries' ability to release an egg, which may be fertilized in the remaining tube if it is undamaged.

A resident practitioner in the birthing unit asks the nurse to prepare for a vaginal examination on a client with a low-lying placenta who is in early labor. What is the priority nursing action?

Explaining why a vaginal examination should not be performed

A nurse in the fertility clinic works with couples who have been trying to become pregnant for more than 1 year. How can the nurse help ease the feeling of isolation that infertile couples often experience?

Explore ways to promote communication with family and friends

A client's nipples become sore and tender as a result of her newborn's vigorous suckling. What should the nurse recommend that the mother do to alleviate the soreness? (Select all that apply.)

Expose the nipples to air several times a day. Apply hydrogel pads to the nipples after each feeding.

A client asks the nurse about the use of an intrauterine device (IUD) for contraception. What information should the nurse include in the response? (Select all that apply.)

Expulsion of the device Occasional dyspareunia Risk for perforation of the uterus

A client asks the nurse about the use of an intrauterine device (IUD) for contraception. What information should the nurse include in the response? (Select all that apply.)

Expulsion of the device Occasional dyspareunia Risk for perforation 구멍of the uterus

A nurse teaching a prenatal class is asked why infants of diabetic mothers are larger than those born to women who do not have diabetes. On what information about pregnant women with diabetes should the nurse base the response?

Extra circulating glucose causes the fetus to acquire fatty deposits.

Which parts of a newborn's body are usually affected by the rash erythema toxicum neonatorum? Select all that apply.

Face Trunk Buttocks

A pregnant client with cardiac disease asks a nurse to clarify what she was told about making the birth easier for her. What should the nurse remind her is an option to facilitate birth?

Facilitating the birth with vacuum extraction

A breastfeeding mother asks the nurse how human milk compares with cow's milk. How should the nurse respond?

Fat in human milk is easier to digest and absorb than the fat in cow's milk.

During a class for prepared childbirth, the nurse teacher discusses the importance of the spurt of energy that occurs before labor. Why is it important to conserve this energy?

Fatigue may influence pain medication requirements.

During a class for prepared childbirth, the nurse teacher discusses the importance of the spurt of energy that occurs before labor. Why is it important to conserve 아끼다this energy?

Fatigue may influence pain medication requirements.

A newborn experiences a hypothermic period while being bathed and having clothing changed. Once the hypothermic episode has been identified and treated, what is the next nursing action?

Feeding the infant A newborn who experiences a hypothermic episode responds by becoming hypoglycemic; providing calories will increase the blood glucose level.

What findings occur with supine hypotensive syndrome? (Select all that apply.)

Feeling of faintness Increased venous pressure Decreased systolic pressure

A pregnant client is admitted to the high-risk unit with uterine tenderness and some dark-red vaginal bleeding. Abruptio placentae is diagnosed. What priority evaluation should be included with vital signs, skin color, urine output, and fetal heart rate?

Fundal height It is vital that a baseline measurement of the height of the fundus be obtained, because increasing size is a sign of concealed hemorrhage; with abruptio placentae, bleeding occurs behind the placenta

A primigravida, unsure of the date of her last menstrual period, is told by the nurse that she appears to be at 22 weeks' gestation. What data support this conclusion?

Fundus at the umbilicus The uterus rises gradually to the level of the umbilicus at around 22 to 24 weeks' gestation.

A primigravida client gave birth in a vaginal delivery 24 hours ago. Which findings would be considered normal?

Fundus firm at the umbilicus; moderate lochia rubra; voiding quantity sufficient; colostrum산모의) 초유 present Twenty-four hours after delivery, the fundus is usually at the umbilicus and moderate lochia rubra is expected. Colostrum is present, and the breast milk usually comes in on day 3 after delivery.

After 2 weeks of radiation therapy for cancer of the breast a client experiences some erythema over the area being radiated. The area is sensitive but not painful. She states that she has been using tepid water and a soft washcloth when cleansing the area and applying an ice pack three times a day. What does the nurse conclude from this information?

Further teaching on skin care is necessary.

Using the five-digit system, determine the obstetric history in this situation: The client is 38 weeks into her fourth pregnancy. Her third pregnancy, a twin gestation, ended at 32 weeks with a live birth, her second pregnancy ended at 38 weeks with a live birth, and her first pregnancy ended at 18 weeks.

G4, T1, P1, A1, L3 Four pregnancies = G (gravida ) 4. One pregnancy that ended at 38 weeks = T (term) 1. One pregnancy that ended at 32 weeks = P (preterm) 1. One pregnancy that ended at 18 weeks = A (abortion) 1. One set of twins and a singleton = L (living) 3.

While a client is being interviewed on her first prenatal visit she states that she has a 4-year-old son who was born at 41 weeks' gestation and a 3-year-old daughter who was born at 35 weeks' gestation and lost one pregnancy at 9 weeks and another at 18 weeks. Using the GTPAL system, how would you record this information?

G5 T1 P1 A2 L2 The client is gravida (G) 5: the current pregnancy, the 41-week pregnancy, the 35-week pregnancy, the 9-week pregnancy, and the 18-week pregnancy. She has had one term (T) pregnancy (one that lasts 40 weeks plus or minus 2 weeks): the 41-week pregnancy. The 35-week pregnancy is considered preterm (P). Pregnancies that end before 20 weeks are considered abortions, so the losses at 9 and 18 weeks would be scored as A2

A pregnant client is making her first antepartum visit. She has a 2-year-old son born at 40 weeks, a 5-year-old daughter born at 38 weeks, and 7-year-old twin daughters born at 35 weeks. She had a spontaneous abortion 3 years ago at 10 weeks. How does the nurse, using the GTPAL format, document the client's obstetric history?

G5 T2 P1 A1 L4

A parent of a newborn asks, "Why do I have to scrub my baby's formula bottles?" What information about a newborn should the nurse consider before replying in language that the parent will understand?

Gastric acidity is low and does not provide bacteriostatic protection.

After being transported to the hospital by the ambulance, a pregnant woman is brought into the emergency department on a stretcher. The nurse notes that the fetus' head has emerged. How should the nurse assist the mother in the birth of the fetus' anterior shoulder?

Gently guiding the head downward

What does the nurse conclude is related directly to an infant's survival in the neonatal period?

Gestational age and birth weight

A primigravida at 8 weeks' gestation is visiting the prenatal clinic for the first time. What should an examination reveal at this time?

Goodell's sign

Which factor in a client's history increases the risk for osteoporosis?

Prolonged immobility

A client in labor, who is at term, is admitted to the birthing room. The fetus is in the left occiput posterior position. The client's membranes rupture spontaneously. What observation requires the nurse to notify the practitioner?

Greenish amniotic fluid Greenish amniotic fluid indicates the presence of meconium and should be reported to the health care provider. The interval between contractions should shorten as labor progresses. Clear fluid with specks of mucus is the description of normal amniotic fluid. There may be a slight increase in temperature related to the stress of labor, and it should be monitored.

A primigravida is admitted to the birthing unit in active labor. The fetus is in a breech presentation. What physiological response does the nurse expect during this client's labor?

Greenish-tinged amniotic fluid Greenish amniotic fluid is common in a breech presentation because the contracting uterus exerts pressure on the fetus' lower colon, forcing the expulsion of meconium

What prenatal teaching is applicable for a client who is between 13 and 24 weeks' gestation?

Growth of the fetus, personal hygiene, and nutritional guidance

The four essential components of labor are powers, passageway, passenger, and psyche. Passageway refers to the bony pelvis. What type of pelvis is considered the most favorable for a vaginal delivery?

Gynecoid A gynecoid pelvis is considered most favorable for a vaginal birth because the inlet allows the fetus room to pass. The gynecoid pelvis is considered the typical female pelvis. An android pelvis, which has a heart shape, is considered a male pelvis. The fetus often gets stuck. The anthropoid pelvis is elongated, with a roomy anterior posterior dimension and a narrower transverse diameter than the gynecoid pelvis. Although delivery is possible with this type of pelvis, it is less likely to be successful. The platypelloid pelvis is flat, with a compressed oval shape as the middle opening, instead of an open circle like the gynecoid pelvis. This is a rare type of pelvis.

A married 35-year-old client is to undergo a tubal ligation. The factor in the history that contributes most to the healthy resolution of any emotional problem associated with sterilization is that the client:

Has a son and daughter and feels that her family is complete

A nurse expects signs of respiratory distress syndrome (RDS) in a neonate whose mother:

Has type 1 diabetes

A nurse expects signs of respiratory distress syndrome (RDS) in a neonate whose mother:

Has type 1 diabetes Infants of diabetic mothers are at risk for respiratory distress syndrome as a result of delayed synthesis of surfactant caused by a high serum level of insulin.

A neighbor who is a nurse is called on to assist with an emergency home birth. What should the nurse do to help expel the placenta?

Have the mother breastfeed the newborn Suckling will induce neural stimulation of the posterior pituitary gland, which in turn will release oxytocin and cause uterine contractions.

◀ A nurse is caring for the newborn of a drug-addicted mother with suspected cytomegalovirus disease. What does the nurse suspect was the cause of the disease?

Having sex with many partners

The nurse is teaching a prenatal breathing and relaxation class. What does the nurse suggest to best ease back discomfort during labor?

Having support persons use back massage techniques

A client with preeclampsia is admitted to the labor and birthing suite. Her blood pressure is 130/90 and she has 2+ protein in her urine and edema of the hands and face. Which signs or symptoms are suggestive that HELLP syndrome is developing? (Select all that apply.)

Headache Abdominal pain Flulike symptoms

A client is admitted to the birthing suite with a blood pressure of 150/90 mm Hg, 3+ proteinuria, and edema of the hands and face. A diagnosis of severe preeclampsia is made. What other clinical findings support this diagnosis? (Select all that apply.)

Headache Abdominal pain Visual disturbances

How should a nurse direct care for a client in the transition phase of the first stage of labor?

Helping the client maintain control

A health care provider prescribes carboprost (Hemabate) to be administered to a postpartum client with intractable vaginal bleeding. What client factor should alert the nurse to question the prescription?

History of asthma Carboprost (Hemabate) is contraindicated in clients with asthma because of its respiratory side effects of coughing and dyspnea. The Homan sign indicates thrombophlebitis and is not related to carboprost. Carboprost does not affect blood pressure. Absence of the Babinski reflex is an expected adult finding.

A nurse is planning for the discharge of a crack-addicted 17-year-old mother and her newborn. What is the most appropriate referral to meet the mother's and infant's needs?

Home health nurse

A nurse identifies a right cephalhematoma on an otherwise healthy 1-day-old newborn. What should the nurse teach the parents at the time of discharge?

How to monitor their child for signs of jaundice Bilirubin is a yellow pigment derived from the hemoglobin released with the breakdown of red blood cells as the hematoma resolves. Signs of jaundice should be reported. The other actions are not specific for a healthy neonate with a cephalhematoma.

A primigravida at 12 weeks' gestation complains of nausea and vomiting during a visit to the prenatal clinic. Which pregnancy hormone should the nurse explain is thought to be responsible for nausea and vomiting during the first trimester?

Human chorionic gonadotropin (hCG)

A client measuring at 18 weeks' gestation visits the prenatal clinic stating that she is still very nauseated and vomits frequently. Physical examination reveals a brown vaginal discharge and a blood pressure of 148/90 mm Hg. What condition does the nurse suspect the client is experiencing?

Hydatidiform mole

A client measuring at 18 weeks' gestation visits the prenatal clinic stating that she is still very nauseated and vomits frequently. Physical examination reveals a brown vaginal discharge and a blood pressure of 148/90 mm Hg. What condition does the nurse suspect the client is experiencing?

Hydatidiform mole A hydatidiform mole, in which chorionic villi degenerate into grapelike vesicles, is the cause of these signs and symptoms.

During a routine prenatal visit, a client tells a nurse that she gets leg cramps. What condition does the nurse suspect, and what suggestion is made to correct the problem?

Hypocalcemia; increase her intake of milk

A small-for-gestational-age (SGA) newborn who has just been admitted to the nursery has a high-pitched cry, appears jittery, and exhibits irregular respirations. What complication does the nurse suspect?

Hypoglycemia

A nurse is caring for a client in labor who is receiving epidural anesthesia. For which common side effect of this route of anesthesia should the client be monitored?

Hypotensive episodes Epidural anesthesia creates a sympathetic block that causes loss of peripheral vascular resistance and a decrease in venous return; this leads to a reduced cardiac output, which can precipitate hypotensive episodes.

Three days after birth, a breastfeeding newborn becomes jaundiced. The parents bring the infant to the clinic and blood is drawn for an indirect serum bilirubin determination, which reveals a concentration of 12 mg/dL. The nurse explains that what the infant has is physiological jaundice, a benign condition, caused by:

Immature liver function

A nurse elicits the Babinski reflex on a newborn. The nurse concludes that this finding indicates:

Immaturity of the central nervous system (CNS)

A nurse suspects that a newborn has toxoplasmosis, one of the TORCH infections. How and when may it have been transmitted to the newborn?

In utero through the placenta Toxoplasmosis is caused by a parasitic protozoon that is acquired from inadequately cooked contaminated food or through handling of infected cat feces; the most common form of transmission to the newborn is by way of placental perfusion when in utero.

A client making her first visit to the prenatal clinic asks which immunization can be administered safely to a pregnant woman. What should the nurse tell her?

Inactive influenza

A client making her first visit to the prenatal clinic asks which immunization can be administered safely to a pregnant woman. What should the nurse tell her?

Inactive influenza The inactive influenza and diphtheria, tetanus, pertussis (dTAP) can be safely administered during the first trimester of pregnancy, although dTAP is recommended at 27 to 36 weeks' gestation to provide immunity to the mother and infant.

A client who wishes to postpone having children until she and her husband are financially sound tells the nurse she has been taking oral contraceptive pills for several years. What finding indicates a potential risk in regard to continued use of birth control pills?

Increased blood pressure The estrogen and/or progesterone in birth control pills increases the amount of renin produced in the kidneys. Increased renin will stimulate the production of angiotensin, a potent pressor substance, resulting in hypertension.

Which adaptation does the nurse suspect is the result of early decompensation in a pregnant woman with cardiac problems?

Increasing fatigue

A hysterectomy is scheduled for a client with endometrial cancer. Before the surgery, what should the nurse prepare the client to expect?

Indwelling urinary catheter A catheter decompresses the bladder and limits trauma to the surgical site; it eliminates the need for repeated straight catheterizations after surgery.

A nurse caring for a pregnant woman determines that she is engaging in the practice of pica. Why should the nurse prepare a teaching plan for this client?

Inedible items are being ingested. Pica is the eating of inedibles such as starch or dirt. There is a cultural influence on this practice, but it may also be related to malnutrition or anemia.

A client at 10 weeks' gestation phones the prenatal clinic to report that she is experiencing some vaginal bleeding and abdominal cramping. The nurse arranges for her to go to the local hospital. The vaginal examination reveals that her cervix is dilated 2 cm. What diagnosis should the nurse expect?

Inevitable abortion Once cervical dilation has begun, the abortion is classified as inevitable. In septic abortion the cervix is dilated and there is bleeding; also, the discharge is malodorous. Bleeding and cramping may be present, but the cervix is still closed in a threatened abortion. The products of conception have been partially expelled in an incomplete abortion.

A client at 10 weeks' gestation phones the prenatal clinic to report that she is experiencing some vaginal bleeding and abdominal cramping. The nurse arranges for her to go to the local hospital. The vaginal examination reveals that her cervix is dilated 2 cm. What diagnosis should the nurse expect?

Inevitable abortion Once cervical dilation has begun, the abortion is classified as inevitable. In septic abortion the cervix is dilated and there is bleeding; also, the discharge is malodorous. Bleeding and cramping may be present, but the cervix is still closed in a threatened abortion. The products of conception have been partially expelled in an incomplete abortion.

A neonate born at 36 weeks' gestation, weighing 2043 g (4 lb 8 oz), is placed under a radiant warmer. An infusion of D10% 0.2 NS is running through an umbilical vein catheter at a rate of 12 mL/hr. Why is it important for the nurse to check the neonate's voidings for specific gravity?

Infants under open radiant warmers are at risk for dehydration.

A nurse who is admitting a newborn to the nursery observes a fetal scalp monitor site on the scalp. For what complication should the nurse monitor this newborn?

Infection

A nurse who is admitting a newborn to the nursery observes a fetal scalp monitor site on the scalp. For what complication should the nurse monitor this newborn?

Infection The monitor site represents a break in the integrity of the scalp, which allows access by microorganisms.

A client's membranes ruptured 20 hours before admission. The client was in labor for 24 hours before giving birth. For which postpartum complication is she at risk?

Infection When the membranes rupture, microorganisms from the vagina may travel into the embryonic sac, causing chorioamnionitis . The longer the time between the rupture of the membranes and the birth, the greater the risk for infection.

☆A client who is at 10 weeks' gestation returns for her second prenatal visit. She asks why she has to urinate so often. The nurse tells her that urinary frequency in the first trimester is:

Influenced by the enlarging uterus, which is still within the pelvis

A pregnant client with an infection tells the nurse that she has taken tetracycline (Tetracyn) for infections on other occasions and prefers to take it now. The nurse tells the client that tetracycline is avoided in the treatment of infections in pregnant women because it:

Influences the fetus's teeth buds

A nurse is interviewing a female client with a tentative diagnosis of cystitis pending laboratory results. The nurse anticipates that the causative agent of the cystitis is Escherichia coli. The nurse anticipates this microorganism because it:

Inhabits the intestinal tract E. coli is commonly found in the bowel and, because of anatomical proximity and possibly careless hygiene after bowel movements, may spread to the urethra.

Shortly after birth a newborn is found to have Erb's palsy. What condition does the nurse suspect caused this problem?

Injury to brachial plexus during birth Erb's palsy is caused by forces that alter the alignment of the arm, shoulder, and neck; stretching or pulling away of the shoulder from the head during birth damages the brachial plexus.

A 23-year-old woman comes to the clinic for a Pap smear. After the examination, the client confides that her mother died of endometrial cancer 1 year ago and says that she is afraid that she will die of the same cancer. Which risk factor stated by the client after an education session on risk factors indicates that further teaching is needed?

Late-onset menarche Early-onset, not late-onset, menarche is a risk factor for endometrial cancer. A high-fat diet, hypertension, and obesity are all risk factors.

A client in preterm labor is to receive a tocolytic medication, and bedrest is prescribed. Which position should the nurse suggest that the client maintain while on bedrest?

Lateral

A pregnant client at 37 weeks' gestation is taught about signs and symptoms that should be reported immediately to the primary care provider. The nurse determines that the client understands the information presented when she states that she will immediately report:

Leakage of fluid from the vagina

A pregnant client at 37 weeks' gestation is taught about signs and symptoms that should be reported immediately to the primary care provider. The nurse determines that the client understands the information presented when she states that she will immediately report:

Leakage of fluid from the vagina Leakage may indicate rupture of the amniotic membranes; the client is at risk for an ascending infection from the vagina if birth does not occur within 24 hours or if early treatment is not instituted.

What should the nurse discuss with new parents to help them prepare for infant care?

Learning specific behaviors involving states of wakefulness to promote positive interactions

What should the nurse recommend to a new mother when teaching her about the care of the umbilical cord area?

Leave the area untouched or clean with soap and water, then pat it dry.

A woman with an active lifestyle is in her 30th week of pregnancy. Which activity will the nurse discourage?

Leg lifts and sit-ups

A woman with an active lifestyle is in her 30th week of pregnancy. Which activity will the nurse discourage?

Leg lifts and sit-ups Maintaining the supine position can interrupt blood flow to the fetus, so this should be avoided after the first trimester.

A woman with an active lifestyle is in her 30th week of pregnancy. Which activity will the nurse discourage?

Leg lifts and sit-ups Maintaining the supine position can interrupt blood flow to the fetus, so this should be avoided after the first trimester. Yoga, swimming, and bicycling are all appropriate activities during pregnancy.

A client is admitted to the high-risk prenatal unit with the diagnosis of placenta previa. What should the nurse instruct the client to do?

Lie on her side to avoid putting pressure on the vena cava

A nurse is caring for a client in labor whose cervix is dilated 6 cm. The client is receiving epidural analgesia. What common response to regional anesthesia does the nurse anticipate?

Lightheadedness

A nurse is caring for a client in labor whose cervix is dilated 6 cm. The client is receiving epidural analgesia. What common response to regional anesthesia does the nurse anticipate?

Lightheadedness Lightheadedness may indicate hypotension resulting from the vasodilation commonly associated with epidural analgesia .

A client who had tocolytic therapy 자궁수축억제제 치료 for preterm labor is being discharged. What instructions should the nurse include in the teaching plan?

Limit daily activities.

During discharge teaching a client who just had a hysterectomy states, "After this surgery, I don't expect to be interested in sex anymore." What should the nurse consider before responding?

Many women incorrectly equate hysterectomy with loss of libido.

Thirty minutes after a client gives birth, the nurse palpates the client's uterus. It is relaxed and the lochia is excessive. What is the nurse's initial action?

Massage the uterus

A nurse who is caring for a mother and her newborn infant reviews their record. In light of the data the record contains, what nursing intervention is required? Type RH: A negative Rubella titer: 1:2 RPR/VDRL: negative HB Sag: negative HIV: negative Hgb/Hct: 11/33 Sickle prep: negative

Maternal rubella vaccination A rubella titer of 1:2 is inadequate immunization. A titer of 1:8 is considered immunity. Rubella immunization protects the fetuses of future pregnancies from significant birth defects caused by a rubella infection.

A pregnant client is experiencing nausea and vomiting. The nurse determines that this discomfort:

May be related to an increased human chorionic gonadotropin level

A client comes to the fertility clinic for hysterosalpingography using radiopaque contrast material to determine whether her fallopian tubes are patent. When preparing for the test, the nurse explains to the client that she:

May have some persistent shoulder pain for 14 hours after the test

A nurse helps a client to the bathroom to void several times during the first stage of labor. This is done because a full bladder:

May inhibit the progress of labor

What does the nursing care for an infant with necrotizing enterocolitis (NEC) include?

Measuring abdominal girth every 2 hours Prolonged gastric emptying occurs with NEC; an increase in abdominal girth of greater than 1 cm in 4 hours is significant and requires immediate intervention.

The day after a hysterectomy, the client asks for sanitary pads because she feels that she is going to menstruate. What information should influence the nurse's response?

Menstruation will not occur because the uterus has been removed.

A multipara is admitted to the birthing room in active labor. Her temperature is 98° F (36.7° C), pulse 70 beats/min, respirations 18 breaths/min, and blood pressure 126/76 mm Hg. A vaginal examination reveals a cervix that is 90% effaced and 7 cm dilated with the vertex presenting at 2+ station. The client is complaining of pain and asks for medication. Which medication should be avoided because it may cause respiratory depression in the newborn?

Meperidine (Demerol) Meperidine (Demerol) is an opioid that can cause respiratory depression in the neonate if administered less than 4 hours before birth. Naloxone (Narcan) is an opioid antagonist that reverses the effects of respiratory depression in the newborn. Lorazepam (Ativan) is a sedative; it does not cause respiratory depression in the newborn, but it does not relieve pain by itself. Promethazine (Phenergan) is a tranquilizer; it does not cause respiratory depression in the newborn. Promethazine does not relieve pain by itself.

After a newborn has skin-to-skin contact with the mother, a nurse places the newborn under a radiant warmer. What complication is the nurse attempting to prevent?

Metabolic acidosis Uncorrected cold stress increases anaerobic glycolysis, which increases acid production, resulting in metabolic acidosis. Metabolic acidosis, not metabolic alkalosis, occurs when a neonate is stressed by cold.

A nursing instructor provides education for the students on thermoregulation in the nursery. The students determine that in the healthy full-term neonate, heat production is accomplished by:

Metabolism of brown fat Metabolism of brown fat releases energy and increases heat production in the newborn.

A nurse is caring for a client who has contracted a trichomonal infection. Which oral drug should the nurse anticipate that the health care provider will most likely prescribe?

Metronidazole (Flagyl) Metronidazole (Flagyl) is a potent amebicide. It is effective in eradicating the protozoan Trichomonas vaginalis. Penicillin is administered for its effect on bacterial, not protozoal, infections. Gentian violet is a local anti-infective that is applied topically; it may cause discoloration of the skin. It is effective against Candida albicans. Nystatin (Mycostatin) is an antifungal for infections caused by C. albicans.

The nurse is obtaining a health history from a client with endometriosis. What consequences can occur as a result of this disorder? (Select all that apply.)

Metrorrhagia Bowel strictures Voiding difficulties

A newborn has small, whitish, pinpoint spots over the nose that are caused by retained sebaceous secretions. When documenting this observation, a nurse identifies them as:

Milia Milia are common, are not indicative of illness, and eventually disappear. Lanugo is fine, downy hair. Whiteheads are a lay term for milia; the term is not used in documentation. Mongolian spots are bluish-black areas on the buttocks that may be present on dark-skinned infants.

A nurse teaches a pregnant woman about the need to increase her intake of complete proteins. Which foods identified by the client indicate that the teaching is effective? (Select all that apply.)

Milk Eggs Cheese

What is the priority nursing care after an amniocentesis?

Monitoring for signs of uterine contractions

At 37 weeks' gestation a client's membranes spontaneously rupture but she does not have contractions. What action is most important in the nursing plan of care for this client?

Monitoring for the presence of fever The possibility of an ascending infection increases when membranes have ruptured and birth is not imminent; the client must be monitored for signs of infection

At 37 weeks' gestation a client's membranes spontaneously rupture but she does not have contractions. What action is most important in the nursing plan of care for this client?

Monitoring for the presence of fever The possibility of an ascending infection increases when membranes have ruptured and birth is not imminent; the client must be monitored for signs of infection.

A client who is in the first trimester is being discharged after a week of hospitalization for hyperemesis gravidarum. She is to be maintained at home with rehydration infusion therapy. What is the priority nursing activity for the home health nurse?

Monitoring the client for signs of electrolyte imbalances

A client at 6 weeks' gestation who has type 1 diabetes is attending the prenatal clinic for the first time. The nurse explains that during the first trimester insulin requirements may decrease because:

Morning sickness may lead to decreased food intake. Morning sickness , a common occurrence during pregnancy, contributes to decreased food intake; the insulin dosage must be reduced to prevent hypoglycemia.

The parents of a newborn with phenylketonuria (PKU) ask a nurse how to prevent future problems. What must the nurse consider before responding?

Most important is the institution of a corrective formula soon after birth.

A nurse understands the stages of parental adjustment that follow the birth of an at-risk infant who is in the neonatal intensive care unit. To better plan nursing care, the nurse bases observations and assessments on the recognition that the:

Mother should be reunited with her infant as soon as possible to enhance adjustment

Where is the best area for the nurse to determine adequate tissue oxygenation in a neonate born of black parents?

Mucous membranes of the mouth

When reviewing the history of a client admitted in preterm labor during her 30th week of gestation, the nurse suspects a risk factor associated with this client's preterm labor. What is this risk factor?

Multiple urinary tract infections

A nurse in the family planning clinic reviews the health history of a sexually active 16-year-old girl whose chief concern is a thick, burning discharge accompanied by a burning sensation and lower abdominal pain. After an examination the girl is informed that she may have a sexually transmitted infection (STI) that requires treatment. The adolescent is concerned that her parents will discover that she has been sexually active and asks the nurse whether her parents will be contacted. The nurse explains that her parents will:

Not be contacted, because treatment at the clinic is confidential

A newborn's hands and feet are cyanotic and there is circumoral pallor when the infant cries or feeds. What should the nurse do?

Notify the practitioner, because circumoral pallor may indicate cardiac problems

A nurse in the newborn nursery is monitoring an infant for jaundice related to ABO incompatibility. What blood type does the mother usually have to cause this incompatibility?

O Mothers with type O blood have anti-A and anti-B antibodies that are transferred across the placenta

A client who has breast cancer had postlumpectomy chemotherapy and is now scheduled for radiation on an outpatient basis. What is an important nursing intervention while the client is receiving radiation?

Observing the irradiated site daily for redness or irritation

During the postpartum period a nurse determines that a client's rubella titer is negative. What action should the nurse plan to take?

Obtaining a prescription for immunization at discharge

A client at 40 weeks' gestation is admitted to the birthing unit in early active labor. She tells the nurse that her membranes ruptured 26 hours ago. Assessments of the fetal heart rate range between 168 and 174 beats/min. What is the priority nursing action?

Obtaining maternal vital signs

An expectant couple asks the nurse about the cause of low back pain during labor. The nurse replies that this pain occurs most often when the fetus is positioned:

Occiput posterior

In specific situations gloves are used to handle newborns whether or not they are HIV positive. When is it unnecessary for the nurse to wear gloves while caring for a newborn?

Offering a feeding

A woman in labor arrives at the birthing unit. She tells the nurse, "They told me the last time I was at the clinic that I had chlamydia, but I stopped taking the antibiotic after 3 days because I felt better." What potential neonatal disorder transmitted during birth most concerns the nurse because of the inadequate treatment?

Ophthalmia neonatorum

The nurse observes that 12 hours after birth the neonate is hyperactive and jittery, sneezes frequently, has a high-pitched cry, and is having difficulty suckling. Further assessment reveals increased deep tendon reflexes and a diminished Moro reflex. What problem does the nurse suspect?

Opioid drug withdrawal

The nurse observes that 12 hours after birth the neonate is hyperactive and jittery, sneezes frequently, has a high-pitched cry, and is having difficulty suckling. Further assessment reveals increased deep tendon reflexes and a diminished Moro reflex. What problem does the nurse suspect?

Opioid drug withdrawal These signs are indicative of withdrawal from an opioid with typical changes occurring in the central nervous system; the newborn should be monitored during the first 24 to 48 hours

What nursing action is the priority for a client in the second stage of labor?

Promote effective pushing by the client

A client with a large fetus is to have a pudendal 외음부(外陰部)의 block during the second stage of labor. What does the nurse plan to instruct the client about the effectiveness of the block? (Select all that apply.)

Perineal pain will not be felt. The bearing-down reflex will be diminished. The block provides anesthesia to the perineum, after which pain is not felt. Although the bearing-down reflex is diminished, muscle control is not affected and the client is able to bear down with contractions. The block affects only the perineum, not the bladder

A client who is having a difficult labor is found to have cephalopelvic disproportion. Which medical order should the nurse question?

Piggyback another 10-unit bag of oxytocin (Pitocin).

A client who is having a difficult labor is found to have cephalopelvic disproportion. Which medical order should the nurse question?

Piggyback another 10-unit bag of oxytocin (Pitocin). When there is cephalopelvic disproportion, a cesarean birth is indicated; infusing oxytocin (Pitocin) at this time could result in fetal compromise and uterine rupture. The nothing-by-mouth (NPO) status is appropriate in anticipation of a cesarean birth. A peripheral IV is needed not only for hydration but also for venous access if IV medications become necessary. The client probably has an electronic monitor recording the fetal heart rate and uterine contractions; the findings of these assessments should be documented regularly in accordance with hospital protocol.

The nurse in the postpartum unit is teaching self-care to a group of new mothers. What color does the nurse teach them that the lochial discharge will be on the fourth postpartum day?

Pinkish brown

A client with frank vaginal bleeding is admitted to the birthing unit at 30 weeks' gestation. The admission data include blood pressure of 110/70 mm Hg, pulse of 90 beats/min, respiratory rate of 22 breaths/min, fetal heart rate of 132 beats/min. The uterus is nontender, the client is reporting no contractions, and the membranes are intact. In light of this information, what problem does the nurse suspect?

Placenta previa A nontender uterus and bright-red bleeding are classic signs of placenta previa ; as the cervix dilates, the overlying placenta separates from the uterus and begins to bleed.

A nurse plans to weigh a newborn. What is the most appropriate way to obtain the newborn's weight?

Placing the naked infant on the scale

The nurse is reassessing a newborn who had an axillary temperature of 97° F (36° C) and was placed skin to skin with the mother. The newborn's axillary temperature is still 97° F (36° C) after 1 hour of skin-to-skin contact. Which intervention should the nurse implement next?

Placing the newborn under a radiant warmer in the nursery

The nurse is reassessing a newborn who had an axillary temperature of 97° F (36° C) and was placed skin to skin with the mother. The newborn's axillary temperature is still 97° F (36° C) after 1 hour of skin-to-skin contact. Which intervention should the nurse implement next?

Placing the newborn under a radiant warmer in the nursery The newborn's temperature should be kept in the normal range of 97.7° F to 99.5° F (36.5° C to 37.5° C ).

A postpartum client is being prepared for discharge. The laboratory report indicates that she has a white blood cell (WBC) count of 16,000/dL. What is the next nursing action?

Placing the report in the client's record because this is an expected postpartum finding Leukocytosis (15,000-20,000 mm3 WBC) typically occurs during the postpartum period as a compensatory defense mechanism. There is no need for further intervention, because the client is exhibiting an expected postpartum leukocytosis.

A strict vegetarian (vegan) becomes pregnant and asks the nurse whether there is anything special she should do in regard to her diet during pregnancy. What is most the important measure for the nurse to instruct the client to take?

Plan to eat from specific groups of vegetable proteins each day.

At 5 am, 2 hours after a long labor and vaginal birth, a client is transferred to the postpartum unit. What is the nurse's priority when planning morning care for this client?

Planning nursing care activities that provide time for the client to rest and sleep

A nurse is caring for a new mother who has a chlamydial infection. Which complications are associated with chlamydial infections in neonates? (Select all that apply.)

Pneumonia Preterm birth Conjunctivitis

A pregnant client asks a nurse for information about toxoplasmosis during pregnancy. What should the nurse teach the client about how to prevent the transmission of toxoplasmosis?

Pork and beef should be cooked well before being eaten.

A client at 40 weeks' gestation is admitted to the birthing unit in labor. During the initial examination the nurse uses Leopold maneuvers to palpate the abdomen. The purpose of this intervention is to assess the:

Position of the fetus

The nurse teaches a client who is to undergo amniocentesis that ultrasonography will be performed just before the procedure to determine the:

Position of the fetus and the placenta

A client with heart disease is admitted to the birthing suite. How can the nurse try to prevent the development of cardiac decompensation during her labor?

Positioning her on the side with her shoulders elevated. The side-lying position, particularly the left, takes the weight off large blood vessels , increasing blood flow to the heart; elevating the shoulders relieves pressure on the diaphragm. The client's head is too low in this position if she is only positioned with her head on a pillow; it should be elevated above the shoulders

Why is it important for the nurse to know the infant's gestational age and how it compares with the birthweight?

Potential problems may be identified.

During labor the nurse encourages the client to void periodically. The nurse knows that an over distended urinary bladder during labor can:

Predispose the client to uterine hemorrhage after birth

A nurse weighs a neonate who is born at 29 weeks' gestation. The weight is 1619 g (3 lb 9 oz). In light of this weight and gestational age, how should this infant be classified?

Preterm

A nurse is testing a newborn's heel blood for the level of glucose. Which newborn does the nurse anticipate will experience hypoglycemia? (Select all that apply.)

Preterm infant Small-for-gestational-age infant Large-for-gestational-age infant

Women who become pregnant for the first time at a later reproductive age (35 years or older) are at risk for what complications? (Select all that apply.)

Preterm labor Multiple gestation Chromosomal anomalies Bleeding in the first trimester

Which observations are suggestive of postmaturity in a newborn male? (Select all that apply.)

Profuse scalp hair Parchmentlike skin Creases covering the entire soles Profuse scalp hair is associated with a postterm newborn. As the fetus matures, usually the hair on the scalp becomes more profuse. Parchmentlike skin is associated with a postterm newborn. Skin desquamation occurs as a result of prolonged exposure to amniotic fluid, causing cracking, peeling, and drying of skin and resulting in a parchmentlike appearance. Creases will cover the entire sole of each foot if the newborn is full term or postterm; preterm newborns have an absence of or few skin creases on the soles of the feet.

A primigravida in the first trimester tells a nurse that she has heard that hormones play an important role in pregnancy. Which hormone should the nurse tell the client maintains pregnancy?

Progesterone Progesterone is a female sex hormone, produced by the ovaries and placenta, that prepares the endometrium for implantation of the fertilized ovum, maintains pregnancy, and plays a role in the development of the mammary glands.

A client is admitted to the birthing unit in active labor. Amniotomy is performed by the health care provider. What physiological change does the nurse expect to occur after the procedure?

Progressive dilation and effacement

A multipara whose membranes have ruptured is admitted in early labor. Assessment reveals a breech presentation, cervical dilation of 3 cm, and fetal station at -2. For what complication should the nurse assess when caring for this client?

Prolapse of the umbilical cord A breech presentation results in a larger space between the cervix and the fetal sacrum than does a vertex presentation.

A client who is in preterm labor at 34 weeks' gestation is receiving intravenous tocolytic therapy. The frequency of her contractions increases to every 10 minutes, and her cervix dilates to 4 cm. The infusion is discontinued. Toward what outcome should the priority nursing care be directed at this time?

Promotion of maternal and fetal well-being during labor Labor is continuing, and promotion of the well-being of both client and fetus is the priority nursing care during this period. Reduction of anxiety associated with preterm labor, supportive communication with the client and her partner, and helping the family cope with the impending preterm birth each address just one aspect of this client's needs and must be dealt with in the context of the priority need.

What antidote to the side effects of terbutaline (Brethine) should a nurse have available?

Propranolol (Inderal) Propranolol (Inderal) is a beta-blocking agent that reverses the uterine inhibitory responses and cardiovascular effects of terbutaline (Brethine). Levodopa (L-dopa) is not an antidote for terbutaline; it is used in Parkinson's disease. Furosemide (Lasix) is a diuretic; it will not reverse the cardiovascular effects indicated. Ritodrine (Yutopar) may cause responses similar to those of terbutaline; it is sometimes used to halt premature labor because it inhibits beta2- receptors.

A nurse is teaching a breastfeeding client about medications that are safe and unsafe for her to take. Which medication is contraindicated?

Propylthiouracil (PTU) The concentration of propylthiouracil (PTU) excreted in breast milk is three to 12 times higher than its level in maternal serum; this may cause agranulocytosis or goiter in the infant

After an uneventful pregnancy a client gives birth to an infant with a meningocele. The neonate has 1-minute and 5-minute Apgar scores of 9 and 10, respectively. What is the priority nursing care for this newborn?

Protecting the sac with moist sterile gauze Preventing infection and trauma is the priority; rupture of the sac may lead to meningitis. The Apgar scores are 9 and 10 at 1 and 5 minutes, respectively; oxygen is not needed.

What should be included in the teaching plan for the mother of a newborn with exstrophy of the bladder?

Protecting the skin surrounding the exposed bladder

A nurse is teaching a class of expectant parents about nutritional needs during pregnancy. What information should the nurse include?

Protein needs increase to at least 70 g/day during pregnancy.

A nurse is discussing immunizations needed to confer active immunity with a pregnant client during her first visit to the prenatal clinic. What information should the nurse consider including that the client will understand with regard to active immunity?

Protein substances are formed by the body to destroy or neutralize antigens.

A client is admitted with a diagnosis of preeclampsia. What significant clinical finding does the nurse expect when reviewing the client's history?

Proteinuria

A client at 36 weeks' gestation arrives at the prenatal clinic for a routine examination. The nurse determines that the client's blood pressure has increased from 102/60 to 134/88 mm Hg and becomes concerned she may be experiencing mild preeclampsia. What other sign of mild preeclampsia does the nurse anticipate?

Proteinuria of 1+

What characteristics cause the nurse to suspect that a newborn has Down syndrome? (Select all that apply.)

Protruding tongue Epicanthal eye folds One transverse palmar crease

What clinical findings does the nurse expect to observe in a newborn with trisomy 21 (Down syndrome)? (Select all that apply.)

Protruding tongue Hypotonic muscle tone Broad nose with a depressed bridge

A client at 36 weeks' gestation is admitted to the high-risk unit because she gained 5 lb in the previous week and there is a pronounced increase in blood pressure. What is the initial intervention in the client's plan of care?

Providing a dark, quiet room with minimal stimuli Increasing cerebral edema may predispose the client to seizures; therefore stimuli of any kind should be minimized.

A newborn is admitted to the neonatal intensive care unit with a myelomeningocele located at the fourth lumbar vertebra (L4). What is the priority nursing intervention while the infant is awaiting surgery?

Providing meticulous skin care

A new father tells the nurse that he is anxious about not feeling like a father. What is the priority nursing action to meet this father's needs?

Providing time for the father to be alone with and get to know the baby

Identify the position of the fetus whose buttocks are in the fundus, whose fetal back is on the maternal right side between the midline, and lateral surface of the abdomen, and whose attitude is general flexion.

ROA The fetus is in the ROA (right occiput anterior) position: occiput facing the front on the right side of the mother). It is a vertex delivery. In the RSA (right sacrum anterior) position the buttocks point anteriorly on the mother's right side. RMA (right mentoanterior) is a brow presentation. In LOA (left occiput anterior), another vertex position, the fetus' back is on the mother's left side.

A nurse is providing dietary counseling to a client at 14 weeks' gestation. The client is a recent immigrant from Asia, and the nurse explores the foods that the client usually eats. Which foods should the nurse counsel the client to avoid during pregnancy? (Select all that apply.)

Raw shellfish Herbal supplements Soft-scrambled eggs

While caring for a client in labor, a nurse notes that during a contraction there is a 15-beat/min acceleration of the fetal heart rate above the baseline. What is the nurse's next action?

Record the fetal response to contractions and continue to monitor the heart rate

An infant born in the 36th week of gestation weighs 4 lb 3 oz (2062 g) and has Apgar scores of 7 and 9. What nursing actions will be performed on the infant's admission to the nursery? (Select all that apply.)

Recording of vital signs Evaluation of the neonate's health status Supportive measures to keep the neonate's body temperature stable

At a routine monthly visit, while assessing a client who is in her 26th week of gestation, the nurse identifies the presence of striae gravidarum. The nurse describes this condition as:

Reddish streaks on the abdomen and breasts

An 18-year-old primigravida at 36 weeks' gestation is admitted with a diagnosis of mild preeclampsia. What is the nurse's most important goal for this client?

Reducing her blood pressure

After a difficult birth, a neonate has an Apgar score of 8 after 5 minutes. Which signs met the criteria of 2 points? (Select all that apply.)

Reflex irritability: cry Respiratory rate: good cry Heart rate: 110 beats/min A cry for reflex irritability rates a score of 2. A good cry for respiratory rate scores a 2. A heart rate of 100 beats/min or more rates a 2. A pink body with blue extremities rates a 1. Some flexion of extremities rates a 1 for muscle tone.

A fetal monitor is applied to a client in labor. The nurse should take action in response to a fetal heart rate that:

Repeatedly drops abruptly to 90 beats/min unrelated to contractions

A client who recently gave birth is transferred to the postpartum unit by the nurse. What must the nurse do first to avoid a charge of abandonment?

Report the client's condition to the responsible staff member

A nurse is counseling a pregnant woman with type 1 diabetes. What is the most important nursing consideration in the planning of care for this client?

Requirement of intensive prenatal care

A client who is at risk for seizures as a result of severe preeclampsia is receiving an IV infusion of magnesium sulfate. What findings cause the nurse to determine that the client is showing signs of magnesium sulfate toxicity? (Select all that apply.)

Respirations of 10/min Loss of patellar reflexes

When a preterm newborn requires oxygen, the nurse in the neonatal intensive care unit monitors and adjusts the oxygen concentration. What complication do these adjustments attempt to prevent?

Retinopathy of prematurity Retinopathy of prematurity is caused by the high concentration of oxygen that may have to be used to support some preterm neonates; oxygen must be administered cautiously and, depending on the neonate's blood oxygen level, adjusted accordingly.

Rho(D) immune globulin (RhoGAM) is prescribed for an Rh-negative client who has just given birth. Before giving the medication, the nurse verifies the newborn's Rh factor and reaction to the Coombs test. Which combination of newborn Rh factor and Coombs test result confirms the need to give Rho(D) immune globulin ?

Rh positive with a negative Coombs result

Before the administration of Rho(D) immune globulin (RhoGAM) the nurse reviews the laboratory data of a pregnant client. Which blood type and Coombs test result must a pregnant woman have to receive RhoGAM after giving birth?

Rh-negative and Coombs negative

A nurse is planning to administer Rhogam (Rh immune globulin). Which situation requires the administration of this medication?

Rh-negative woman who has had an amniocentesis

A newborn has congenital cataracts, microcephaly, deafness, and cardiac anomalies. Which infection does the nurse suspect that the newborn's mother contracted during her pregnancy?

Rubella Congenital rubella (German measles) syndrome results in abnormalities that vary, depending on the gestational age of the fetus when the maternal infection was contracted; the most severe results occur if the mother was infected during the first trimester, when organogenesis is taking place. Neonatal signs of herpes virus type 2 include fever, coryza, tachycardia, and hemorrhage. Except for microcephaly, the assessments noted by the nurse are not caused by the toxoplasmosis protozoa; this problem is associated with growth retardation, hydrocephalus, chorioretinitis, thrombocytopenia, jaundice, and fever. A chlamydial infection causes neonatal conjunctivitis and pneumonia.

A nurse is preparing a client with a ruptured tubal pregnancy for immediate surgery. What type of surgery should the informed consent include?

Salpingectomy

A client appears at the clinic after getting a positive result on a home pregnancy test. She states that her last menstrual period began 10 weeks ago. The client expresses fear because she has been recently found to have syphilis. What prescriptions will the nurse expect to receive from the health care provider because of this information? (Select all that apply.)

Screening and testing of sexual partners Intramuscular benzathine penicillin G, 2.4 million units, one dose

A client appears at the clinic after getting a positive result on a home pregnancy test. She states that her last menstrual period began 10 weeks ago. The client expresses fear because she has been recently found to have syphilis. What prescriptions will the nurse expect to receive from the health care provider because of this information? (Select all that apply.)

Screening and testing of sexual partners Intramuscular benzathine penicillin G, 2.4 million units, one dose CDC) recommends the intramuscular administration of 2.4 million units of benzathine penicillin G in one dose for those, pregnant or nonpregnant, in whom syphilis has been diagnosed within the preceding year. Syphilis is a bacterial Infection; usually spread by sexual contact. This treatment has been found effective for pregnant women and newborns. Diagnosis of syphilis and no allergy to penicillin should be verified before medication is administered. Sexual partners should be screened and tested to prevent the spread of syphilis. Untreated syphilis can result in stillbirth, preterm birth, and birth defects.

A pregnant woman at 6 week's gestation tells the nurse at her first prenatal visit that she uses an over-the-counter herbal product as a health supplement that has been approved by the Food and Drug Administration. What should the nurse recommend to the client? (Select all that apply.)

Stop taking the supplement immediately. Discuss the use of the supplement with the practitioner. Discuss the use of any over-the-counter products with the practitioner.

The nurse is caring for a client whose labor is to be induced. What is the nurse's responsibility when a client's labor is being stimulated with an oxytocin (Pitocin) infusion?

Stopping the infusion if contractions become hypertonic

A nurse observes a laboring client's amniotic fluid and decides that it is the expected color. What finding supports this conclusion?

Straw-colored, clear, and containing little white specks

A client states that she wishes to use the calendar method of birth control. The nurse concludes that the client understands how to calculate the beginning of the fertile period when she states, "I will:

Subtract 18 days from the length of my shortest cycle"

A client states that she wishes to use the calendar method of birth control. The nurse concludes that the client understands how to calculate the beginning of the fertile period when she states, "I will:

Subtract 18 days from the length of my shortest cycle" The fertile period is determined by subtracting 18 days from the length of the shortest cycle to determine the first unsafe day and subtracting 11 days from the length of the longest cycle to determine the last unsafe day.

A nurse is planning to teach a new mother about breastfeeding. What should the nurse consider before preparing the client to breastfeed?

Suckling stimulates the release of oxytocin.

During a vertex vaginal birth the nurse notes meconium-stained amniotic fluid. What is the priority nursing intervention for the newborn?

Suctioning the airway

What clinical manifestation requires immediate intervention in a woman with a probable ruptured tubal pregnancy?

Sudden onset of knifelike pain in one of the lower quadrants

What is the best nursing action for a client in active labor whose cervix is dilated 4 cm and 100% effaced with the fetal head at 0 station?

Suggest that the client accept the PRN medication for pain that has been prescribed

Which position does the nurse teach the client to avoid when she experiences back pain during labor?

Supine position

Supporting the parents

Supporting the parents

During assessment of a newborn, a practitioner diagnoses cephalhematoma and informs the parents. The mother asks why her baby's head looks different. What does the nurse take into consideration before responding in terms that the mother will understand?

Swelling that is confined to one part of the scalp is caused by hemorrhage beneath the periosteum. A cephalhematoma occurs during labor when the rim of the pelvis exerts pressure on the fetal occiput, causing bleeding between the cranial bone and the periosteum; the hematoma does not cross the suture line.

The nurse is teaching a sex education course to high school students. What should the nurse teach them about why gonorrhea is difficult to control? (Select all that apply.)

Symptoms of the disease are vague. The incubation period is relatively short. Causative organisms have become resistant to treatment.

A client at 28 weeks' gestation with previously diagnosed mitral valve stenosis is being evaluated in the clinic. Which sign or symptom indicates that the client is experiencing cardiac difficulties?

Syncope on exertion Syncope on exertion is a definitive sign of cardiac decompensation ; cardiac output is not meeting cellular oxygen needs.

A client at 28 weeks' gestation with previously diagnosed mitral valve stenosis is being evaluated in the clinic. Which sign or symptom indicates that the client is experiencing cardiac difficulties?

Syncope on exertion is a definitive sign of cardiac decompensation ; cardiac output is not meeting cellular oxygen needs.

The gravida 1 now para 1 woman delivered a 7-lb 6 oz female infant at 11 pm yesterday after a labor of 14 hours. After breakfast the nursery staff brings the baby to the new mother. The mother smiles at the baby, then asks that the nurse take the baby back to the nursery because she has not had a shower yet. One hour later the nurse returns with the infant. Again the mother smiles at the baby; then she holds her, kisses her, and feeds her a bottle. Immediately after feeding the baby, the mother calls the nursery and asks that the baby be picked up so she can take a nap. What behavior is the new mother demonstrating?

Taking-in During the taking-in period the mother focuses on her needs rather than the baby's.

A nurse is teaching a childbirth preparation class about the discomfort of labor. What is the greatest influence on the perception of pain for a woman in labor?

Tension of the client Tension, which prevents relaxation, has the greatest influence on how a woman will progress through labor and on her perception of pain.

What should the care of a newborn infant whose mother has had untreated syphilis since the second trimester of pregnancy include?

Testing for congenital syphilis

A nurse caring for a client who gave birth to a healthy neonate evaluates the client's uterine tone 8 hours later. How does the nurse determine that the uterus is demonstrating appropriate involution?

The amount of lochia rubra is moderate.

A primipara gives birth to an infant weighing 9 lb 15 oz (4508 g). During labor a midline episiotomy is performed and the client sustains a third-degree laceration. The client tells the nurse that her perineal area is very painful. What should the nurse consider before explaining the reason for the pain?

The anal sphincter muscle has been injured.

The parents of a newborn are concerned about red pinpoint dots on their infant's face and neck. How should the nurse explain the finding?

The cause is an increased intravascular pressure during birth.

A nurse is caring for a client during the transition phase of labor. The nurse determines that the client has entered the second stage of labor when:

The client reports that she feels the urge to move her bowels

A client who is visiting the prenatal clinic for the first time has a serology test for toxoplasmosis. What information about the client's activities in the history indicates to the nurse that there is a need for this test?

The client takes care of a cat.

A client's membranes rupture spontaneously during the latent phase of the first stage of labor, and the fluid is greenish brown. What does the nurse conclude?

The fetus may be compromised in utero.

A client's membranes rupture spontaneously during the latent phase of the first stage of labor, and the fluid is greenish brown. What does the nurse conclude?

The fetus may be compromised in utero. Greenish-brown amniotic fluid is a sign of meconium in utero, which may indicate that the fetus is compromised.

A nurse is caring for a client at 42 weeks' gestation who is having a contraction stress test (CST). What does a positive result indicate?

The function of the placenta has diminished.

A nurse is caring for a client at 42 weeks' gestation who is having a contraction stress test (CST). What does a positive result indicate?

The function of the placenta has diminished. During a CST uterine blood flow to the placenta decreases. When a decrease is too great, fetal hypoxia and late decelerations occur, reflecting diminished placental function.

An internal fetal monitor is applied while a client is in labor. What should the nurse explain about positioning while the monitor is in place?

The most comfortable position may be assumed.

A client who has had a mastectomy asks what the term ERP-positive means. How should the nurse explain this finding?

The tumor cells generally exhibit a positive response to hormone therapy that reduces estrogen.

A nurse caring for a pregnant client and her partner suspects domestic violence. Which observations support this suspicion? (Select all that apply.)

The woman has injuries to the breasts and abdomen. The partner answers questions that are asked of the woman. The woman has visited the clinic several times in the last month.

A contraction stress test (CST) is performed on a client at 40 weeks' gestation. The findings are interpreted as negative. What does the nurse conclude from this interpretation?

There will be weekly retesting because, at this time, the fetus has oxygen reserves.

2. ★ Quiz: Pregnancy, Labor, Childbirth, Postpartum - At Risk A contraction stress test (CST) is performed on a client at 40 weeks' gestation. The findings are interpreted as negative. What does the nurse conclude from this interpretation?

There will be weekly retesting because, at this time, the fetus has oxygen reserves. A negative test result implies that placental support is adequate; it is associated with a low fetal death rate within 1 week.

Continuous positive-pressure ventilation therapy by way of an endotracheal tube is started in a newborn with respiratory distress syndrome (RDS). The nurse determines that the infant's breath sounds on the right side are diminished and that the point of maximum impulse (PMI) of the heartbeat is in the left axillary line. How should the nurse interpret these data?

These findings indicate that the infant may have a pneumothorax, and the health care provider should be contacted immediately.

What does the nurse expect concerning the alveoli in the lungs of a 28-week-gestation neonate?

They have a tendency to collapse with each breath.

A new mother's laboratory results indicate the presence of cocaine and alcohol. Which craniofacial characteristic indicates to the nurse that the newborn has fetal alcohol syndrome (FAS)? (Select all that apply.)

Thin upper lip Small upturned nose Smooth vertical ridge in the upper lip

A client at 7 weeks' gestation tells a nurse in the prenatal clinic that she is sick every morning with nausea and vomiting and adds that she does not think she can tolerate it throughout her pregnancy. The nurse assures her that this is a common occurrence in early pregnancy and will probably disappear by the end of the:

Third month

A client at 12 weeks' gestation arrives in the prenatal clinic complaining of cramping and vaginal spotting. A pelvic examination reveals that the cervix is closed. Which probable diagnosis should the nurse expect?

Threatened abortion

During a newborn assessment the nurse counts the infant's cord vessels. What does the nurse expect to observe in a healthy newborn?

Three vessels: one vein and two arteries

Laboratory studies reveal that a pregnant client's blood type is O and she is Rh-positive. Problems related to incompatibility may develop in her infant if the infant is:

Type A or B An ABO incompatibility may develop even in firstborn infants because the mother has antibodies against the antigens of the A and B blood cells; these antibodies are transferred across the placenta and produce hemolysis of the fetal red blood cells; if the infant were AB, an incompatibility might also occur.

A client has a diagnosis of an unruptured tubal pregnancy. Which findings correlate with this diagnosis? (Select all that apply.)

Unilateral abdominal pain History of a sexually transmitted infection

A preterm infant is started on digoxin (Lanoxin) and furosemide (Lasix) for persistent patent ductus arteriosus. Which clinical finding provides the best indication of the effectiveness of the furosemide?

Urine output exceeds fluid intake.

A nurse is caring for a client who is being given intravenous magnesium sulfate to treat preeclampsia. Which adverse side effect alerts the nurse to notify the health care provider?

Urine output of less than 100 mL in 4 hours A decreased urine output of less than 25 mL/hr may be indicative of kidney damage, a result of the preeclampsia, and impending renal failure. Magnesium sulfate is excreted by the kidneys, and magnesium toxicity may occur.

A client is receiving an epidural anesthetic during labor. For which side effect should the nurse monitor the client?

Urine retention

What should the nurse do when an apnea monitor sounds an alarm 10 seconds after cessation of respirations?

Use tactile stimuli on the chest or extremities The nurse applies tactile stimulation after confirming that respirations are absent; this action may be sufficient to reestablish respirations in the high-risk neonate with frequent episodes of apnea.

For what complication should the nurse specifically monitor a grand multipara who has just given birth?

Uterine atony

For what complication should the nurse specifically monitor a grand multipara who has just given birth?

Uterine atony Grand multiparas have diminished uterine muscle tone as a result of the repeated distentions of pregnancy; consequently, the uterine muscles may not contract effectively during the fourth stage of labor.

What complication should a nurse be alert for in a client receiving an oxytocin (Pitocin) infusion to induce labor?

Uterine tetany

A nurse places fetal and uterine monitors on the abdomen of a client in labor. While observing the relationship between the fetal heart rate and uterine contractions, the nurse identifies four late decelerations. What condition is most frequently associated with late decelerations?

Uteroplacental insufficiency Late decelerations , suggestive of fetal hypoxia, occur in the setting of uteroplacental insufficiency.

At 38 weeks' gestation a client is admitted to the birthing unit in active labor, and an external fetal monitor is applied. Late fetal heart rate decelerations begin to appear when her cervix is dilated 6 cm and her contractions are occurring every 4 minutes and lasting 45 seconds. What does the nurse conclude is the cause of these late decelerations?

Uteroplacental insufficiency Late decelerations are indicative of uteroplacental insufficiency and, left uncorrected, lead to fetal hypoxia, fetal myocardial depression, or both.

A client gives birth vaginally, with a midline episiotomy, to an infant who weighs 8 lb 13 oz (4000 g). An ice pack is applied to the perineum to ease the swelling and pain. The client complains, "This pain in my vaginal and rectum is excruciating, and my vagina feels so full and heavy." What does the nurse suspect as the cause of the pain?

Vaginal hematoma

Supplemental oxygen is ordered for a preterm neonate with respiratory distress syndrome (RDS). What action does the nurse take to reduce the possibility of retinopathy of prematurity?

Verifying oxygen saturation frequently to adjust flow on the basis of need Determining oxygen saturation identifies the need for oxygen supplementation; prolonged use of oxygen concentrations exceeding those required to maintain adequate oxygenation contributes to the occurrence of retinopathy of prematurity.

A client who is taking an oral contraceptive calls the nurse with concerns about side effects of the medication. Which adverse effect of this medication should alert the nurse to inform the client to immediately stop the contraceptive and contact the health care provider? (Select all that apply.)

Visual disturbances Persistent headaches Visual disturbances, such as partial or complete loss of vision or double vision, may indicate neuro-ocular lesions, which are associated with the use of oral contraceptives. Persistent headaches may indicate hypertension, which may occur with the use of contraceptives. Nausea is an expected side effect and does not require notification of the HCP. Weight gain, not weight loss, may occur because of edema. The client may experience hypertension, not hypotension.

A nurse instructs a client who is taking oral contraceptives to increase her intake of dietary supplements. Which supplement should be increased?

Vitamin C

A nurse instructs a client who is taking oral contraceptives to increase her intake of dietary supplements. Which supplement should be increased?

Vitamin C Oral contraceptives can affect the metabolism of certain vitamins, particularly vitamin C, and supplementation may be required

▶ The nurse assures a breastfeeding mother that one way she will know that her infant is getting an adequate supply of breast milk is if the infant gains weight. What behavior does the infant exhibit if an adequate amount of milk is being ingested?

Voids six or more times a day

While observing a newborn, the nurse notes that the skin is mottled. What should the nurse do first?

Warm the environment Mottling results from hypothermia; the newborn should be wrapped, placed under a radiant warmer, or given to the mother for skin-to-skin contact.

A nurse is teaching a breastfeeding mother about cleansing her nipples. What technique should the nurse emphasize?

Wash the breasts and nipples with water when bathing.

A mother and her newborn have just been transferred to the postpartum unit from labor and delivery. What infant safety education should be provided? (Select all that apply.)

Wash your hands before touching the newborn. All client identification bands should remain in place until discharge. Check the identification of staff and if there is a question of validity, call the nursing station.

A client in labor is receiving an oxytocin (Pitocin) infusion. For which adverse reaction resulting from prolonged administration should the nurse monitor the client?

Water intoxication

A client in labor is receiving an oxytocin (Pitocin) infusion. For which adverse reaction resulting from prolonged administration should the nurse monitor the client?

Water intoxication Oxytocin (Pitocin) has an antidiuretic effect, acting to reabsorb water from the glomerular filtrate.

What should the nurse teach a formula-feeding mother about breast engorgement when it occurs?

Wear a tightly fitted brassiere.

A primigravida at 34 weeks' gestation tells the nurse that she is beginning to experience some lower back pain. What should the nurse recommend that the client do? (Select all that apply.)

Wear low-heeled shoes. Perform pelvic tilt exercises several times a day.

A client with endometriosis asks the nurse what side effects to expect from leuprolide (Lupron). What should the nurse include in the response?

Weight gain

A client with endometriosis asks the nurse what side effects to expect from leuprolide (Lupron). What should the nurse include in the response?

Weight gain The nurse should teach the client that the side effects of leuprolide (Lupron) include edema, which causes an increase in weight. Leuprolide decreases libido. Frequent urination is not a side effect of leuprolide. Clients who take leuprolide do not experience menstrual periods, because follicle-stimulating hormone and luteinizing hormone are suppressed.

What is the priority nursing intervention during the 2 hours after a cesarean birth?

What is the priority nursing intervention during the 2 hours after a cesarean birth?

A nurse is teaching a class about childbearing and contraceptive options. The nurse explains that fertilization of the ovum by the sperm occurs at a specific time. When does it occur?

When one sperm penetrates the wall of the ovum

When is it most important for a female client to know that a fetus may be structurally damaged by the ingestion of drugs?

When she is planning to become pregnant

What client behavior indicates to the nurse that a woman needs further teaching about breastfeeding her newborn?

When she leans forward to place her breast in the infant's mouth

What client behavior indicates to the nurse that a woman needs further teaching about breastfeeding her newborn?

When she leans forward to place her breast in the infant's mouth When the breast is pushed into the infant's mouth, typically the infant's mouth closes too soon, resulting in inadequate latching on.

A client is to undergo a tuberculin test as part of her prenatal workup. Before administering the test, what information about the client should the nurse obtain?

Whether an earlier tuberculin test's result was positive

A client is admitted in preterm labor. After intravenous tocolytic 자궁수축억제제 medications are administered, contractions cease and she is discharged. She is to receive oral terbutaline (Brethine) 5 mg every 6 hours at home. When should the nurse advise the client to take the medication?

With food One side effect of terbutaline (Brethine) is nausea and vomiting; to minimize this problem it should be ingested with food.

List the mechanisms of labor in the correct sequence:

engagement descent flexion internal rotation extension restitution external rotation expulsion

A nurse is providing nutritional counseling to a low-income pregnant client who has iron-deficiency anemia. What food should the nurse encourage the client to include in her diet each day to best address this problem?

½ cup of red kidney beans


Ensembles d'études connexes

Macro Final: Short Answers and HWK 4

View Set

Cisco Computer Support & Service (CSS)

View Set

Mark Twain and Regionalism Study Guide

View Set

Strategic Management - Chapter 6

View Set

Chapter 10: Assessment of Aptitude

View Set

WB 3.2 - Government: Enlightenment influences

View Set